Medicine Long Cases - DR - Praveen

Download as pdf or txt
Download as pdf or txt
You are on page 1of 118

A GUIDE FOR MEDICAL STUDENTS

PRAVEEN WEERATUNGA
Prolonged fever

Introduction to the patient

Background – highlight the relevant aspects of the medical background of the patient

Presenting complaint

Presentation is with prolonged fever

Description of the presenting complaint

 Describe the presenting complaint in a chronological sequence


 Describe the onset and progression of the illness
 Try to describe the clinical pattern of the fever and the basic symptoms associated with it
However, it is important to note that the classical fever pattern is commonly altered due to
the use of antibiotics and antipyretics
 Describe the symptoms associated with the height of the fever and those in the fever free
periods

Fever patterns relevant to a case of prolonged fever

See Diagram 1

Analysis and further questions

 A proper chronology is essential before


progressing
 The first step is to establish the clinical profile
of the patient. This will determine the
subsequent part of the history
 Determine whether the patient is,
From the community
Has a recent history of hospitalization
Is Immunocompromised – on
immunosuppressant drugs or long-term steroids? Diagram 1
 The focus is likely to be on a patient from the community, presenting with
prolonged fever
 Think of the differential diagnosis and ask specific questions
 The aetiological agents causing PUO are classified as,
Infection
Neoplastic
Autoimmune
Other – Endocrine, drugs
 Once you have established the temporal profile of the presentation, proceed to ask direct
and more specific questions
Category Diagnosis Specific questions
Infections
Bacterial Abscess formation and Liver – consumers of alcohol, right hypochondrial
localized pyogenic pain, Pelvic - pelvic symptoms – PV discharge
infection Also ask for localized joint pain and bone pain
(septic arthritis and osteomyelitis)

Infective endocarditis Past history of valvular or congenital heart disease,


cardiac surgical procedures such as insertion of
prosthetic valves or devices
Associated features of heart failure

Tuberculosis Nocturnal fever with associated chills and night


sweats. Associated history of chronic cough,
hemoptysis or shortness of breath, other features
of extra pulmonary involvement – backache, SI
joint pain

Brucellosis Ask for exposure to animals, consumption of


unpasteurized milk, testicular pain, respiratory
symptoms, associated back ache

Typhoid Ask for associated alteration of bowel habits, the


classical progression in typhoid commences with
constipation and subsequent diarrhea

Typhus Recent visit to Southern, Northern or North


Central province of Sri Lanka, associated joint and
muscle pain, skin rash

Melioidosis High risk occupation – from a farming community,


associated skin and parotid abscesses or multiple
deep abscesses

Infections Retroviral infection Ask in detail on high risk sexual exposures,


Viral intravenous drug use and travel history
Ask for a past history of recurrent sino-pulmonary
infections
A patient with possible retroviral infection may
have fever due to infection with opportunistic
pathogens
Infections Malaria Recent history of foreign travel to Africa or other
Parasitic malaria prevalent countries
Bowel parasites Chronic diarrhoea
Inflammatory - SLE History of inflammatory joint pain, oral ulcers,
autoimmune alopecia, skin rashes
Other associated symptoms – especially urinary –
hematuria, persistent frothy urine, edema or CNS
– episodes of altered level of consciousness or
seizures
Vasculitis
Giant cell arteritis – ask for typical temporal
headache, jaw claudication while eating, shoulder
girdle pain

Still’s disease
Inflammatory type joint pain, transient rash
Neoplastic Lymphoma Predominantly nocturnal fever, with associated
night sweats
Significant loss of weight
Pruritus
Ask if the patient has felt neck, axillary or inguinal
lumps (lymph nodes)
Chronic leukaemia Associated features of anaemia, bleeding
manifestations, evidence of recurrent infections

Solid organ
malignancies Abdominal pain and discomfort, hematuria
Renal cell carcinoma
Other Hyperthyroidism Progressive loss of weight despite increased
appetite
Ask for other features such as increased sweating,
irritability
Drugs Ask for recent use of medication
Certain drugs may cause DRESS syndrome and
present with prolonged fever

 After you have done this, establish the patient’s course through healthcare services,
including the investigations and interventions which have been performe d
 This may give you a clue to the diagnosis, but do not be biased in your evaluation based on
this

Past medical and surgical history

Travel history in detail

Social history

 Sexual history
Examination

General examination

Perform a detailed general examination in a patient with prolonged fever

 General state of the patient


 Pallor – chronic disease, hematological malignancy
 Look for tender pulseless temporal arteries – Giant cell arteritis
 Icterus
 Lymphadenopathy – this may suggest – Lymphoma, disseminated infection, retroviral
infection
 Goiter
 Clubbing – IE
 Stigmata of infective endocarditis – Osler’s nodes, Janeway lesions, splinter hemorrhages
 Skin rashes – look for skin rashes typical of autoimmune disease

Cardiovascular examination

 Look for obvious regurgitant murmurs suggestive of infective endocarditis


 An atrial myxoma can mimic endocarditis but auscultation will reveal a mid- diastolic rumble

Respiratory

 Look for features of pleural effusion and lung abscess


Abdomen

 Hepatosplenomegaly – infective (typhoid, melioidosis, HIV), lymphoma, or other


hematological malignancies

Nervous system

 Do not forget to look at the fundus – Roth spots (IE),


Discussion

The most likely question you will be asked is to generate a list of differential diagnosis . Use the
following clinical scenarios, which are commonly encountered in patients with prolonged fever to
give a rational list.

Prolonged fever with a cardiac murmur

 Infective endocarditis
 (Rarely – atrial myxoma)
Prolonged fever with weight loss and organomegaly

 Disseminated TB
 Lymphoma
 Retroviral infection
Prolonged fever with rash and joint pain

 Typhus
 Autoimmune disease -SLE, Stills disease
It is also useful to know the definition of PUO – Some examiners may ask you for it

This is the classical definition of PUO. Several changes have been proposed to the time durations
mentioned, but these are arbitrary.

 Fever higher than 38.3°C (101°F) on several occasions, usually interpreted as at least three
 Illness of more than 3 weeks duration and no diagnosis made after 1 week of inpatient
investigation
 Or three outpatient visits

The discussion will then proceed on the lines of basic investigations and management of the most
likely diagnosis. Remember that FBC, blood picture, inflammatory markers and blood cultures are
essential investigations that you have to offer in any patient with PUO.

Infective endocarditis
Case summary

A 50-year-old woman presented with high grade intermittent fever for 4 weeks duration with
associated anorexia, weight loss and chills. She also complains of exertional dyspnoea, orthopnoea
and paroxysmal nocturnal dyspnoea for 3 days duration. She has a history suggestive of 2 past
episodes of acute rheumatic fever in childhood and has probably not received prophylaxis.

Examination is significant for features of Grade III MR and AR complicated with heart failure
What investigations would you perform
in a patient with suspected endocarditis?

The confirmation of a diagnosis of


infective endocarditis is based on the
modified Duke’s criteria

This requires – positive blood cultures and


suggestive echocardiographic criteria

Echocardiogram

This is a critical investigation in the


diagnosis

Trans - thoracic echocardiogram (TTE) –


first line imaging modality (sensitivity 40-
60%)

If inconclusive and high probability of IE


consider Trans - oesophageal
echocardiography TOE. In TOE the probe
is inserted through the mouth to the
oesophagus which is very close to the
aorta). The TOE has better detection of abscesses and vegetation size, TOE is also better at detecting
prosthetic valve IE

Repeat TOE/TTE in 7-10 if initial investigations negative but clinical probability remains high

Important findings – oscillating mass – vegetations, new valvular regurgitation, abscess, dehiscence
of a prosthetic valve

Microbiological investigations

 The usual recommendation is three sets of blood cultures collected from different
venipuncture sites, with at least 1 h between the first and last draw. The exact criteria for
culture positivity in IE is complicated, but 2 positive cultures from typical endocarditis
organisms is usually sufficient
 These typical micro-organisms are,
 Viridans streptococci, Streptococcus bovis or HACEK group or community-acquired S. aureus
or enterococci, in the absence of a primary focus)
 Negative cultures are seen in patients on antibiotic therapy, HACEK organisms, Coxiella, and
Chlamydia
 If asked why cultures are frequently negative in IE – the most likely cause is prior antibiotic
therapy
 Serological and other test are indicated in patients with culture negative endocarditis

General laboratory investigations – FBC – neutrophil leucocytosis, elevated ESR and CRP, UFR –
microscopic hematuria

 CXR – heart failure


 ECG – conduction blocks in valve ring abscesses
What are the causes of culture negativity in IE?

 Prior antibacterial therapy, fungal endocarditis, other culture negative organisms, improper
technique of collection

Discuss the principles of management of a patient with infective endocarditis

 You should be able to discuss the principles of management as a house officer


Antibiotic therapy

Treatment is with IV antibiotics. The choice of antibiotics can differ

 Based on the setting


 And if the endocarditis is in a native valve or prosthetic valve

In a patient with native valve endocarditis the usual choice as empirical therapy in C. Penicillin and
Gentamicin. This might be changed later with consultation with the microbiology team based on
blood culture sensitivities. Prosthetic valve endocarditis is a complicated problem and requires
coverage for Staphylococci and MRSA. An urgent surgical consult is also required

You should mention that you would commence antibiotics empirically based on clinical suspicion .
The duration of antibiotic therapy depends on the setting, cause and the patient. In some cases , this
could be for up to 6 weeks

Monitoring

 A discussion on how you would monitor the patient is essential


 On a daily ward round,
 Monitor for fever, evidence of heart failure and auscultate the murmur – changing intensity
of the murmur might indicate progression of disease
 Request frequent ECGs in patients with AV endocarditis – this is because infection spreading
around the aortic valve (peri valvular abscess) would cause disruption of the conduction and
AV blocks
 Look out for drug related adverse effects – Renal functions require monitoring in patients on
Gentamicin
 Look for IV - line related adverse effects – These patients require long term antibiotic
therapy. Therefore, a discussion on vascular access is very important

What are the causes for persistent fever in a patient with IE?

 Wrong antibiotics or wrong dose – discuss this with the microbiologist


 Look for local complications of the disease, or rarely; distal septic emboli – emboli in IE can
lodge in the brain, cerebral vessels (mycotic aneurysm) or kidney. Right sided IE can cause
lung abscesses
 Local complications can usually be detected by 2D Echo – valve ring abscesses
 Always think of other concomitant infections such as UTIs and cannula site infections
 In a patient who has clinically improved and inflammatory markers are declining, fever might
be due to a drug fever
When would you consider getting a cardiothoracic surgical opinion?

 Severe valvular dysfunction resulting in heart failure


 Uncontrolled valvular infection
 Large vegetations with embolic phenomenon

What would you do on discharge?

 Refer the patient for cardiology and echocardiographic assessment – this would be to assess
valvular function following completion of antibiotic therapy
 The patient will require antibiotic prophylaxis for endocarditis for invasive dental procedures

Case Summary

A 40-year-old businessman presented with prolonged fever for 3 weeks duration. The symptoms were
more towards the afternoon and associated with night sweats, anorexia and weight loss. He also has
symptoms suggestive of anaemia. Examination is significant for generalized lymphadenopathy and
hepato- splenomegaly.

What is the most likely diagnosis?

Lymphoma

Why did you think of lymphoma in this patient?

 Lymphoma should be considered in the differential diagnosis in a patient with PUO and the
following manifestations
 Constitutional symptoms
 Generalized lymphadenopathy
 Organomegaly
 This is a close differential diagnosis of disseminated TB, sarcoidosis and retroviral infection
(with disseminated TB or lymphoma)

How would you investigate this patient?

 The most important investigation to establish the diagnosis of lymphoma is a histological


confirmation
 If superficial nodes are palpable arrange excision biopsy
 Perform ultrasound scan of the abdomen and CT chest and abdomen to evaluate deep
lymphadenopathy
 USS/CT guided biopsies can be arranged from deep lymph nodes
 Discuss that you will request for bone marrow examination – bone marrow infiltration
 The classical histological appearance of HL is the presence of Reed- Sternberg cells in the
histological specimen
 The histological appearance of NHL varies based on the type
 LDH is more a marker of prognosis rather than diagnosis
A summary on the types of lymphoma and their evaluation are presented below

Hodgkin’s Lymphoma Non-Hodgkin’s Lymphoma


Introduction B cell lymphoma Multiple subtypes
Characteristic cell – Reed Sternberg Can arise from T cells, B cells or NK cells
cell Majority from B cell origin
No Reed Sternberg cells

Examples
B cells
Small lymphocytic lymphoma, mantle cell
lymphoma, follicular lymphoma, marginal zone
lymphoma, Burkitt’s lymphoma

T cells – Adult T cell lymphoma


Aetiology EBV Genetic factors
Genetic Infections – EBV, HTLV1, HHV6, Hep C
Immune deficiency – congenital and acquired
disorders – AIDS, RA, SLE, Sjogren
Exposure to chemicals
Clinical Lymphadenopathy Lymphadenopathy
manifestations Commences in one group and has May be multiple at the time of diagnosis
contiguous spread Noncontiguous spread

Commonly involves nodes above the


diaphragm – cervical, axillary and May have symptoms localized to an organ system
mediastinal – i.e. CNS lymphoma

B symptoms – weight loss, night


sweats, persistent fever B symptoms common
Generalized pruritus – may predate Extranodal spread common
lymphadenopathy Leukaemic phase common
Bone marrow involvement and
Leukaemic phase – rare
Rare – paraneoplastic syndromes
Paraneoplastic syndrome – peripheral
neuropathy, GBS, glomerulonephritis
Investigations Lymph node biopsy Lymph node biopsy – several subtypes –
Reed Sternberg cells histologically and by immunophenotyping
Subtypes identified based on
histology
Staging Based on site of involvement and
presence/absence of B symptoms

Management The mainstay of treatment in HL is The exact mode of management depends on the
with chemotherapy. Radiotherapy type of NHL
may be utilized in patients with
localized disease
Shortness of breath

Introduction to the patient

Background – highlight the relevant aspects of the medical background of the patient

Presenting complaint

 Presentation is with shortness of breath


 In the long case the focus is usually on patients with chronic shortness of breath

Description of the presenting complaint

 Make sure that you do an extensive description of the presenting complaint i n chronological
sequence
 Describe the onset, evolution and progression of the symptom
 Ask about possible exacerbating and relieving factors
 Ask about the associated symptoms

Think of the possible differential diagnosis and ask specific questions. Remember that chronic
shortness of breath can be due to,

 Cardiac disease – cardiac failure, valvular disease


 Pulmonary disease – obstructive airways disease (asthma, COPD), bronchial malignancy
interstitial lung disease, bronchiectasis and pulmonary hypertension
 Renal disease – due to fluid overload and pulmonary oedema
 Anaemia

Diagnosis Specific questions


Cardiac disease

Cardiac failure The pattern of dyspnea is usually on exertion. Ask for associated orthopnea and
paroxysmal nocturnal dyspnea
Look for associated ankle and lower limb swelling
Look for a possible aetiological clue,
Ask for a history suggestive of ischaemic heart disease

Valvular disease Ask for a history suggestive of rheumatic fever in the past
The patient may also complain of intermittent palpitations (suggestive of paroxysmal
atrial fibrillation. Also ask for episodes suggestive of cardiac syncope. This is a common
presenting symptom in patients with aortic stenosis
Respiratory The characteristic pattern of shortness of breath in a patient with asthma is the
disease intermittent and episodic nature of symptoms. The patient is well in between
Asthma exacerbations. Ask for common precipitants which increase symptoms and diurnal
variation of symptoms
Ask for wheezing as the predominant manifestation of shortness of breath
Also inquire on associated allergic rhinitis and eczema and symptoms of GORD. These
commonly coexist in patients with asthma

COPD The characteristic pattern of shortness of breath in COPD is of a progressive nature


with acute exacerbations
COPD patients usually have a baseline level of shortness of breath between
exacerbations
Acute exacerbations are highlighted by cough with productive sputum
A significant history of smoking should also be extracted. In non-smokers, COPD might
occur due to industrial fumes and biomass fuels such as cooking fumes due to
firewood. It is very important to obtain a good history of exposure

Bronchiectasis The predominant symptom in these patients will be cough with profuse sputum
production. This sputum production is often positional and has a very purulent and
offensive smell. Ask for hemoptysis
Ask for past history of tuberculosis and recurrent episodes of respiratory tract
infections in childhood

Interstitial lung The pattern of shortness of breath in these patients is of a progressive nature. They
disease will have associated cough (non- productive)
It is important to ask about a possible aetiological factor
Ask for a history of connective tissue disease – joint pain, skin rashes, alopecia, oral
ulcers, Raynaud’s phenomenon, proximal muscle weakness (immune mediated
myositis)
Look for occupational exposures and use of long - term drugs (amiodarone,
nitrofurantoin)
Ask for exposure to birds and hay (hypersensitivity pneumonitis)

Progressive Infection with opportunistic pathogens may cause progressive shortness of breath. It is
pulmonary therefore important to inquire on risk factors for immunosuppression – drugs, high
infection risk sexual activities, IV drug use

Bronchial Look for associated cough, hemoptysis and constitutional symptoms such as loss of
malignancy appetite and loss of weight. Ask for alteration in the character of the voice

Pulmonary There are no specific points in the history to confirm this


hypertension But ask for past history of recurrent thrombotic episodes, pregnancy losses in females
Renal disease Episodic shortness of breath with associated uremic features such as itching of the
body, nausea, hiccups, vomiting, metallic taste in the mouth
Neurological Neuromuscular weakness can also cause progressive shortness of breath. But this is
disease exceedingly rare
Anaemia A diagnosis of exclusion of the above. Ask for associated lethargy, malaise, poor
exercise tolerance

 After you have done this, establish the patient’s course through healthcare services,
including the investigations and interventions which have been performed
 This may give you a clue to the diagnosis, but do not be biased in your evaluation based on
this
 Evaluate the current functional state of the patient
Family history – state relevant family history

Drug history
Social history

 Re-evaluate the smoking history


 Give a good environmental history
 Assess the impact of the shortness of breath on the patient’s activities of daily living and
instrumental activities of daily living
 Discuss the impact of the symptoms on the patient’s occupation
 Explore the patient’s psychological state and concern on the current symptoms
Examination

General examination

 Look for pallor – anaemia


 Look for Horner syndrome and facial puffiness with distended neck veins (SVC obstruction)
 Plethora – chronic hypoxia
 Facial rashes
 Lymphadenopathy – supraclavicular nodes in patients with bronchial carcinoma
 Examine the hands for clubbing (bronchiectasis, ILD, bronchial carcinoma), nicotine stains,
look for evidence of connective tissue disease (rheumatoid hands, systemic sclerosis)
 Ankle edema – cardiac failure

Cardiovascular examination

 Look for elevated JVP – congestive cardiac failure, cor-pulmonale


 Look for features of cardiac failure such as dilated apex, functional murmurs (MR and TR),
added sounds (S3)

Respiratory

 Perform a detailed RS examination to look for a respiratory cause for shortness of breath.
Refer the section on RS physical sign interpretation for further details

Abdomen

 Look for Organomegaly

Nervous system

 Do a quick screening exam. You will not find much in the nervous system
Discussion on a patient with heart failure

Case summary

Mrs. D, 68 -year-old housewife presented progressive shortness of breath. She has experienced
shortness of breath on exertion over past six months which has increased in severity and during last
week she has experienced shortness of breath when walking about 50m on the ground level. This was
associated with production of pinkish sputum in scanty amounts. Though she denies any diurnal
variation of symptoms, she complains of worsening of difficulty in breathing on lying supine. Over the
recent days wakes up in the night due to episodes of shortness of breath. She is a diagnosed patient
with hypertension for 5 years. She is on medication with poor compliance. She has had one episode of
chest pain 6 months ago which was managed as heart attack.

On examination she is in severe distress. She is propped up and there are evidences for usage of
accessory muscles for respiration. No ankle oedema was noted. Her pulse was regular, low volume at
a rate of 90/min. Apex of the heart was in the 6th ICS lateral to the MCL. No audible murmurs. Her RR
was 28/min and there were diffuse B/L crepitations in the lung fields.

This is a very common discussion topic at the final MBBS. Therefore , a good working knowledge of
heart failure is essential for all medical students.

Why do you say this patient has heart failure?

 Discuss this on the patient’s symptoms and signs with associated risk factors
 Shortness of breath with orthopnoea, PND and ankle edema
 Examination features of a displaced cardiac apex, elevated JVP, functional MR and TR,
features of pulmonary edema
 Ascites and tender hepatomegaly

How would you establish a diagnosis of heart failure using investigations?

The standard mode of diagnosis is the echocardiogram, but the ECG and CXR would be done as initial
investigations

 ECG
Can demonstrate evidence of previous MI, LV hypertrophy, arrhythmias, poor progression of
R waves in the precordial leads may also be noted.
 CXR
This may demonstrate cardiomegaly, upper lobe diversion, Kerley B lines, perihilar
prominence and pleural effusion (R>L)
 Echocardiogram
This is essential for the diagnosis. A routine 2D echocardiogram may demonstrate low EF,
chamber dilation, regional wall motion abnormalities (in patients with ischaemic heart
disease). The presence of a structural cause such as valvular heart disease or
cardiomyopathy can also be seen.
Some patients have clinical evidence of heart failure but have a normal ejection fraction.
These patients have diastolic dysfunction. This can also be assessed using echocardiography.
 Biomarkers
BNP, N terminal pro BNP – elevated (more in patients with a depressed EF). Natriuretic
peptides are substances secreted by the cardiac chambers in response to cardiac disease
and increased stress on the heart. These biomarkers are also used to prognosticate patients
with chronic heart failure

How would you manage this patient?

 The first step in the management is classification. The is based on the following three
systems.
 Based on the ejection fraction

 Based on the stage of heart failure


Stage A – This group includes patients at risk for heart failure. They do not have structural
heart disease or signs and symptoms of heart failure
Stage B – This group includes patients with structural heart disease, but do not have signs or
symptoms of heart failure
Stage C – This group includes patients with symptomatic heart failure
Stage D – Refractory heart failure
 Based on patient symptoms – NYHA – classification

The most likely patient that you will encounter at the long case will be a patient with HF (reduced
ejection fraction) in stage C. The subsequent discussion will focus on the management of this
common scenario. The management of heart failure with preserved EF is beyond medical student
level

General management

 Due to the chronic nature of the disease proper patient education is paramount. The patient
should be educated on the disease, complications, medications and compliance
 Lifestyle modifications such as graded exercise and cessation of smoking should be
promoted
Pharmacological management

Discuss that you will review the first line medication for heart failure as follows

 Diuretic therapy
 Practical points to discuss – mention that you will monitor the electrolytes carefully in these
patients – especially sodium and potassium. Postural hypotension is a problem in the elderly
due to the increased risk of falls
 Frusemide is the most commonly used diuretic
 ACEI and ATII RB
Should be commenced in the patient unless contraindicated (see below). Discuss that you
will commence with a low dose and gradually increase it. Monitor the patient’s potassium
levels and serum creatinine
The medication should be stopped if the serum potassium increases above 5.5
ATII RB should be substituted if the patient develops cough
Mention that you will adjust the dose if systolic blood pressure falls below 90mmHg
 Beta blockers
Bisoprolol, metoprolol and carvedilol are essential drugs in stable heart failure. Watch for
adverse effects, especially bradycardia

In patients with poor response to the above medication and EF < 35% the following medication
should be considered. In your long case the patient will probably fit into this category

 Spironolactone
Discuss that spironolactone can cause hyperkalemia, and that you will monitor the serum
potassium levels very carefully. This is especially important if the patient is on high doses of
ACEI/ATII RB
 Ivabradine
This is a newer evidence - based option that is available in Sri Lanka. To use ivabradine, the
patient should be receiving optimum first line therapy with a heart rate of > 70 and in sinus
rhythm

Other medication
Digoxin

 Reduces hospitalization due to exacerbations of heart failure. Usually not a first line drug
and maybe of use in patients who are refractory to other medication. May be used earlier in
patients with heart failure can concomitant AF. Toxicity is exaggerated with hypokalemia.
Close monitoring is required

ISMN and hydralazine combination – not routinely used but may have a role in patients intolerant to
ACEI and AT-II receptor blockers. Evidence of benefit only in some racial groups

How would you follow up this patient?

 The high risk, difficult to manage patient should be referred to a cardiologist


 Review the symptoms of the patient and the laboratory investigations
 Review the drug list of the patient, making sure that no harmful drugs have been co-
administered (i.e. thiozolidinediones, NSAIDs, diltiazem, verapamil)
Adverse effect profiles of the major drugs used to treat heart failure are given in the table below

Drug Adverse effects Contraindications

Diuretics Postural hypotension

Metabolic disturbance

Hyperglycaemia, Hyperuricaemia,

Hypokalemia

Hyponatremia

Other

Urinary retention

ACEI Known angioedema

Significant renal dysfunction

Hyperkalemia

Bilateral renal artery stenosis

AT-II RB Similar side effect profile apart from Hyperkalemia


cough and angioedema
Significant renal dysfunction

Beta blockers Bradycardia, conduction abnormalities, Bronchial asthma


bronchoconstriction, worsening of
Heart block
peripheral vascular disease, impotence

Spiranolactone Painful gynaecomastia, hyperkalemia

Digoxin Heart block, pre excitation syndromes,


nausea, vomiting, visual disturbances

What are the other options available for management of patients with refractory heart failure?

 In patients with advanced heart failure, there is incoordination in the contractile


mechanisms of the heart. Cardiac resynchronization therapy uses an electric current to re -
establish coordinated cardiac activity
 Arrhythmias are the commonest cause of death in patients with heart failure. Implantable
defibrillator use in selected patients can reduce the mortality
 If you are asked about these devices in your long case, you have probably done very well!
Mention that these devices are expensive and currently used only in selected patients
Bronchial asthma

Mr P, a 21-year-old student who is a diagnosed patient with bronchial asthma for the past 15 years
presenting with acute onset difficulty in breathing associated with cough. He finds it difficult to speak
and finds it difficult to complete one sentence. SOB was acute onset and it has been worsen ing over
time since today evening after he finished cleaning his study room. He finds it difficult to use the inhaler
that he uses to reduce SOB. He denies any chest pain, orthopnoea or haemoptysis.

He Is a no—smoker. He was diagnosed to have asthma at the age of 5 years and has been using the
blue colour inhaler for symptom relief. He has not been on any long -term medication or inhaler
treatment. Though SOB was on exposure to dust and food items such as pineapple in the early days,
now he experiences symptoms almost two to three times per week during day time and about once or
twice a month in the nights. He has been admitted 4 times with exacerbations over the last year but
has never been ventilated or treated in ICU. His father is also an asthma patient and Mr P has had itchy
skin conditions in his lower leg when he was young. He lives in a house near main road and his uncle
smokes inside house. Frequent symptoms impair his day to day activities especially his academic
performance.

On examination an averagely built man in distress and features of SOB such as intercostal recessions,
nasal flaring, purse lip and supra clavicular recession are visible. He does not have pallor, cyanosis or
finger clubbing. His RR is 28/min and there are diffuse rhonchi and few crepitations over B/L lung fields.
No features of pneumothorax. Cardiovascular examination was normal with a pulse rate of 110/min

How would you manage this patient on initial presentation?

 As in other emergencies, perform a quick assessment of the pati ent with the history and a
focused examination
 Connect the patient to a cardiac monitor and measure the saturation
 Your main objectives are to assess the severity of the episode and make sure the patient is
hemodynamically stable

Classification of severity in an exacerbation of bronchial asthma


How would you manage an acute severe exacerbation of BA?

 Immediately start the patient on oxygen (facemask or non rebreathing mask with reservoir
bag for high flow oxygen)
 Start oxygen driven nebulization with 5mg salbutamol – this can be repeated if the patient
does not show an adequate response (15-20 minutes) or even back to back if required
 Start ipratropium bromide 0.5mg via oxygen driven nebulization
 Start IV hydrocortisone 100mg stat and every 6 hours or prednisolone 40mg daily
 Monitor the patient’s clinical parameters and saturation

The patient does not improve with your initial management. What will you do now?

 If the patient does not improve, or develops life threatening features, call for expert opi nion.
Also perform an arterial blood gas and an inward chest x ray to exclude a pneumothorax if
you cannot clinically diagnose one
 Start the patient on IV magnesium sulphate
1.2 – 2g bolus
 You can consider giving IV salbutamol or IV
aminophylline (250mg bolus if the patient is
not on oral theophylline and 750mg over 24
hours as an infusion)
 However, aminophylline is associated with
cardiac arrhythmias and electrolyte
abnormalities and is best avoided
 Consider supporting the patient with ventilation if the patient is not improving, or is getting
exhausted
 You can use non- invasive ventilation if available
 Keep invasive mechanical ventilation as the last resort

Continue to monitor the patient

The patient is improving – what now? – De-escalating therapy

 Look for the cause for the exacerbation and address it


 Convert nebulization to inhaled medication

How would you confirm a diagnosis of bronchial asthma? Does this patient have asthma?

 This is usually a clinical diagnosis. The important features in the history and examination
pointing to a diagnosis of BA are listed below.
 Presentation with respiratory symptoms: wheeze, cough, breathlessness, chest tightness
 Recurrent episodes of symptoms and symptom variability
 Observation of wheeze
 Personal history of atopy
 Confirmation can also be performed with lung function testing (spirometry)/PEFR
Lung function testing in BA

 Reversibility of the obstruction is demonstrated by a 12% increase in FEV1 and / or a 200 ml


increase in FVC, 20 minutes after an inhaled short acting beta 2 agonist (e.g. 200 micrograms
of salbutamol). In clinical practice this is rarely done and simple to use hand held peak
expiratory flow rate (PEFR) measurements are used. Twice daily PEFR may demonstrate the
reversibility of airway obstruction. A difference of more than 20% is strongly in favor of
reversible airway obstruction seen in asthma.

How would you manage this patient in the long term?

 Discuss that you will perform a detailed evaluation of the cause for the exacerbation
 The causes for poor control in BA include the following
Risk exposure and precipitating events – smoking, occupational and environmental
exposures
Treatment related issues – compliance and inhaler technique
Disease progression
Resistant BA
Co-morbidities that can increase symptoms – GORD and OSA
 Explore the above causes and manage them as necessary
 Proper assessment of the inhaler use and advice on the inhaler technique is essential
 Discuss that your treatment is to achieve complete control of the disease. Complete control
is defined as,
No daytime symptoms
No night-time awakening due to asthma
No need for rescue medication
No asthma attacks
No limitations on activity
including exercise
Normal lung function (in practical
terms FEV₁ and/or PEF >80%
predicted or best)
Minimal side effects from
medication

Pharmacological therapy for BA

 The diagram given on the right


describes the management of BA
using pharmacological therapy
 SABAs are used to relieve
symptoms in patients with mild
intermittent disease
 Therapy is then stepped up to
achieve control as in the diagram
 There is a current trend to use
the LABA + ICS (formoterol +
beclomethasone) as both maintenance and reliever therapy (MART) instead of pure
preventer therapy with add-on SABA
 Specialist therapies include monoclonal antibodies that block IgE pathway
Drug Mechanism Clinical use Adverse effects
Beta 2 agonists Activates B2 receptors to SABA Tremors, palpitations,
increase cAMP Rapid onset. Action tolerance
Relax airway smooth muscle for 3-6 hours
cells Used for symptom
Also reduce activation of relief
mast cells and airway
edema LABA – salmeterol
Duration = 12 hours
Use with steroids as
LABA does not
decrease airway
inflammation
Also therapeutic
interaction noted
Useful for patients
with poor response
to inhaled steroids

Anti cholinergics Prevent cholinergic nerve Not as effective as B2Dry mouth, rare –
induced agonists urinary retention and
bronchoconstriction and Slower onset of worsening of
mucus secretion action glaucoma
Theophylline Inhibition of PDE in airway Not as first line Nausea, vomiting,
smooth muscle increasing therapy, use low palpitations
cAMP doses At high concentrations
Also has anti inflammatory – cardiac
effects arrthythmias, seizures,
low toxic/therapeutic
ratio – can be
triggered by drugs
blocking the CYP450
enzyme –
Erythromycin
Smokers require
higher doses
Inhaled Anti inflammatory Most effective Hoarseness, oral
corticosteroids mechanism controller medication candidiasis
Reduce eosinophills and Posterior subcapsular
number of activated T cells cateract in elderly
and mast cells May cause
Have an effect on airway osteoporosis with high
remodeling doses
Antileukotrienes Block cys-LT1 receptors Used as add on
therapy
Aspirin sensitive
asthma
Exercise induced
asthma
Cromones Mast cell stabilzers Useful in exercise Safe but low efficacy
induced asthma
COPD

Mr. P, a 56-year-old laborer was admitted to hospital with acute shortness of breath from which he
has recovered. On admission he was severely breathless, could not talk in complete sentences and
had to be nebulized several times before his condition improved. He has been having similar episodes
at a frequency of once a month and these episodes have started at age of 48. He had noticed his
symptoms worsening when the sputum turns yellow without a diurnal variation. There was no past
history of childhood asthma. There are no episodes of sudden onset shortness of breath at night but
he notes breathlessness on severe exertion. There were no other significant co morbidities, but he
admits to being a heavy smoker (more than 10 per day for 15 years). He had abstained from smoking
over the last 5 years.

On examination he appeared wasted and was propped up in bed. He spoke with difficulty and use of
accessory muscles and pursed lip breathing was noted. There was no lymphadenopathy but bilateral
ankle oedema was observed. The pulse was 80/ min and regular. A parasternal heave was felt on
palpation. There were bilateral rhonchi on auscultation in both lungs with a patch of bronchial
breathing in left lower zone. There was a mildly tender hepatomegaly without splenomegaly on
abdominal examination.

What is your diagnosis?

Acute exacerbation of chronic obstructive pulmonary disease

Why do you say so?

This patient has presented with an acute episode of shortness of breath with a history of recurrent
exacerbations. He also has dyspnoea on exertion. Given the history the most likely differential
diagnosis which should be considered are COPD and asthma. However, the following aspects in the
history are more suggestive of COPD than asthma

 Age of onset 48 years


 Strong history of smoking
 Productive cough and gradually progressive symptoms
Patients with asthma have intermittent and variable symptoms with frequent nocturnal
episodes

Examination shows that he has features of cor pulmonale (ankle edema and tender hepatomegaly)
and evidence of a consolidation in the left lower zone

How will you evaluate this patient?

 The first step is to review the diagnosis of COPD. Discuss that you will review the following
investigations to establish the diagnosis
 CXR - hyperinflation as evidenced by increased
lucency of the lung fields, flattening of the
diaphragm, bullae
 Lung function testing
This will show reduced FEV1/FVC ratio with poor
reversibility with bronchodilators (<12% reversibility
and < 200ml improvement of FEV1), increased total
lung capacity, RV, reduction in diffusing capacity
(Please note that reversibility testing and diffusing
capacity measurement is not routinely done for
diagnosis of COPD)
 ABG - hypoxemia, CO2 retention
 Arrange for investigations to evaluate complications of COPD – pulmonary hypertension and
RHF

How would you manage an exacerbation of COPD?

Initial management

 Start the patient on oxygen, keeping in mind that a target saturation of 88-92% is generally
adequate. Higher oxygen saturation can cause respiratory depression due to elimination of
the hypoxic respiratory drive in these patients
 Start nebulization with 5mg of salbutamol and 0.5mg of ipratropium bromide
 Start the patient on intravenous hydrocortisone (100mg 6 hourly) or on oral prednisolone
(40-50mg)
 Start intravenous antibiotics – in patients who have significant increase in dyspnea, cough,
purulent sputum production and require mechanical ventilation
Co-amoxyclav or a macrolide can be used as initial therapy, but antibiotics with
pseudomonal cover are required if the patient has had recurrent exacerbations in the past

What would you do if the patient is not responding to this initial management?

 Call for expert opinion


 Arrange an arterial blood gas and chest x ray for further evaluation
 Send other basic investigations
 If the patient does not respond to initial nebulization and other measures, one should
consider mechanical ventilation. The common cause for deterioration in these patients is
exhaustion and muscle fatigue
 The arterial blood gas is an important tool in deciding the requirement of mechanical
ventilation
 Ventilation should be considered in patients with hypercapnic respiratory failure (PCO2>45
mm Hg and arterial pH≤7.35).
 Try NIV first if possible
 Invasive mechanical ventilation may be required if the patient if extremely unstable and
cannot tolerate NIV

How will you manage this patient in the long term?

Discuss this on the following themes


Non – pharmacological management

 Advice on smoking cessation


 Pulmonary rehabilitation – this is a very important aspect of COPD management. Even
though there is a lack of access to properly designed pulmonary rehabilitation programs in
Sri Lanka at present. You should mention that you will try to achieve the following outcomes
with involvement of physiotherapy services, the nutrition unit and substance use
management clinics
 Exercise training
 Inspiratory muscle training
 Psychosocial counseling
 Nutritional evaluation and counseling
 Education, including on proper use of prescribed drugs

Vaccination

 Arrange for the Pneumococcal vaccine -especially if more than 65 years of age
 Influenza vaccination
Pharmacological management

The pharmacological management of COPD is complex, but you should remember the following
principles. When deciding the management, the following factors are considered.

 Patient symptoms – patient symptoms are assessed by the mMRC score and the COPD
assessment test (CAT) score
 Frequency and severity of exacerbations

Exacerbations – more and LAMA Options include,


severe Tiotropium LAMA
LAMA/LABA combination
LABA + ICS combination
(especially useful in patients
with high Eosinophil counts)

Advanced COPD patients who


do not respond to above can
be treated with triple therapy
LAMA+LABA+ICS

If increasing exacerbations –
consider macrolides
Exacerbations – few Bronchodilator Long acting bronchodilators
SABA – when required LABA/LAMA
LABA/LAMA
Symptoms – mild Symptoms – moderate -
severe
Long term Oxygen therapy

 This is considered in patients with advanced COPD who are persistently hypoxaemic +/-
evidence of pulmonary hypertension and right heart failure
 Should be administered for more than 15 hours a day in order to have a beneficial effect on
survival
 LTOT cannot be prescribed in patients who continue to smoke – for obvious reasons!!

Interstitial lung disease

Case summary

Mr. B, 61 year old labourer presented with progressive difficulty in breathing on exertion for 3 months
duration. Initially he has noticed SOB while working and it has progressively got worse. Now he
becomes breathless on walking about 50 meters. Over a few months he became breathless even with
mild physical activity. There has been associated cough with whitish sputum production. No history of
haemoptysis. He denies any diurnal variation of symptoms and no complains of orthopnoea or PND.
No associated malaise, LOA or LOWt. No features of any connective tissue disorder. He is a non smoker.
He does not have past history of TB or exposure to long term drugs and chemicals. His past medical,
surgical and family history are insignificant.

On examination he is ill looking and propped up. There is grade II clubbing with B/L ankle oedema. RR
was 20/min and there were fine end inspiratory crepts in both bases. PR was 88/min with elevated JVP.
No other significant findings except left parasternal heave with palpable P2 and loud S2.

What are your reasons for diagnosing interstitial lung disease?

 The diagnosis of ILD should be considered in patients presenting with shortness of breath of
progressive nature, with non- productive cough
 Clubbing with bilateral fine, basal end-inspiratory crepitation on clinical examination
 Presence of connective tissue disorders (RA, SS)

What is the aetiology?

 Discuss the common


aetiological agents of ILD based
on the classification
demonstrated
How would you investigate this patient?

The diagnosis of interstitial lung disease is established based on the following

 Discuss that you will start with a CXR looking for reticulo-nodular infiltrates
 Ask for lung function tests – this will demonstrate the typical restrictive pattern with
reduction in DLCO and KCO (performed at MRI Colombo)

HRCT patterns

 The HRCT is a major tool in the diagnosis of ILDs. The pattern of disease on HRCT may be
diagnostic in some cases

Other investigations

 Discuss that you will evaluate the need for a transbronchial biopsy or open lung biopsy
based on the distribution of the lesions on HRCT
 Arrange a 2D echocardiogram to assess complications – pulmonary hypertension
How would you manage this patient?

 Patient education and counselling


 This is extremely important in patients with some forms of ILD as they have a poor prognosis
 Some ILDs respond to steroids
 Anti-fibrotic medication is used in ILD with prominent fibrosis
Chest pain

Introduction to the patient

Background – highlight the relevant aspects of the medical background of the patient

Presenting complaint

Chest pain

Description of the presenting complaint

 It is important to establish a good analysis of the symptom. The following aspects are of
extreme importance
 Site of the chest pain
 Onset and progression of the pain
 Character of pain
 Radiation of the pain
 Associations of the chest pain
 Timing
 Exacerbating and relieving factors
 Severity of the patient and gradation based on patient perception

Based on the above analysis certain causes of chest pain should be differentiated with confidence

Specific questions
Cardiac chest pain
Ischaemic heart disease The classical features of cardiac chest pain are, central location
of pain, constricting in nature with radiation to the neck, jaw or
upper limbs, associated autonomic symptoms suggesting visceral
origin – nausea, vomiting and regurgitation
Ask for past history of similar episodes and risk factors for
coronary artery disease

Pericarditis Central location of pain, which has a sharp character and a


pleuritic quality

Aortic dissection This causes instantaneous onset pain which has a tearing
character. Radiation is noted through the back

Respiratory disease
Pneumonia Chest pain of pleuritic nature, associated acute shortness of
breath, fever and sputum production

Pulmonary embolism Chest pain can occur due to small pulmonary emboli, ask for
associated pleuritic chest pain, hemoptysis and shortness of
breath. Larger emboli will present with syncopal/pre-syncopal
episodes
Ask for possible risk factors – immobilization, malignancy,
Pneumothorax thrombophilic conditions in the past
Acute onset pleuritic chest pain with worsening shortness of
breath
Ask for a past history of lung disease
Gastro-esophageal reflux Ask for associated epigastric pain, dyspepsia, abdominal
distension
Esophageal spasm
Neurological Ask carefully for a band like sensation of spreading chest pain
This may indicate thoracic nerve root involvement
Also make note of lower limb neurological symptoms
Musculoskeletal Diagnosis of exclusion
The pain is usually localized and alters with movement

In a long case the focus is on ischaemic heart disease

 After you have done this, establish the patient’s course through healthcare services,
including the investigations and interventions which have been performed
 This may give you a clue to the diagnosis, but do not be biased in your evaluation based on
this
 Evaluate the current functional state of the patient
 Also establish the presence of any complications including features suggestive of left
ventricular failure
Past medical history

 Elaborate on past episodes and investigations performed


 Discuss relevant interventions performed
 Discuss on related comorbidities and risk factors – DM, HT, dyslipidaemia

Family history

 Ischaemic heart disease


 Premature coronary disease in childhood and cardiac death

Drug history

 Take a detailed drug history and discuss compliance and side e ffects
Social history

 Discuss on smoking and alcohol use


 Discuss the impact of these symptoms on the patient’s life
Examination

General examination

 Look for pallor


 Look for peripheral stigmata of hypercholesterolemia including xanthelasma, tendon and
eruptive xanthomata, corneal arcus
 Hands for nicotine stains
 Ankle edema

Cardiovascular

 Look for clinical evidence of heart failure


 Measure the blood pressure
Abdomen

 Look for epigastric tenderness and masses

Nervous system

 Perform a quick screening examination

Chest pain

Case summary

Mr. Y a 45-year-old with a background of HT and Type II DM presented with central chest pain on
exertion for 4 weeks duration. The patient describes this pain as a central tightening type pain with
radiation to left upper limb and both jaws. Chest pain occurs exclusively on exertion, of around 200m
walking and never occurs at rest. The patient does not have any associated shortness of breath or
ankle edema. Examination reveals a BP of 170/80; heart sounds are normal. The rest of the
examination is unremarkable

What is the diagnosis?

 The diagnosis is of chronic stable angina (chronic coronary disease)

How would you confirm the diagnosis?

 This diagnosis can usually be made by a proper clinical evaluation and taking a detailed
history of chest pain. If a clear history is obtained further investigations to confirm the
diagnosis are not required.
 When the history is not conclusive investigations may be considered for confirmation of the
diagnosis
 This includes stress testing – Exercise ECG, stress echocardiography
 CT – Coronary angiogram in patients with stable angina
This detects calcium in atherosclerotic plaques and can detect coronary lesions with high
sensitivity. However, the rate of detection of significant obstructive lesions may be poor. CT
CA may have an application as a tool for negative prediction

Management of a patient with chronic stable angina

 Discuss that you will focus your management on the following themes
 Lifestyle modifications and management of other co-morbidities
 Pharmacological management
 Risk stratification and further management

Pharmacological therapy

Antiplatelet therapy

 Aspirin alone and clopidogrel in patients intolerant to aspirin. Combination has no added
benefit unless following ACS/ or following stent implantation.

Statins

 The patient should be started on high dose statins


ACEI

 ACE inhibitors demonstrate survival benefit in patients with IHD, particularly those with DM,
hypertension, early stage CKD and associated heart failure

Antianginal medication

The primary goal of these medications is to reduce cardiac ischaemia and symptoms of angina.
Several classes of drugs are available.

 Nitrates
Short acting preparations such as GTN are used sublingually for immediate relief
Nitrates act via systemic vasodilation with reduction in end diastolic volume and dilation of
epicardial coronary arteries via nitric oxide and subsequent increase in cGMP
Adverse effects – headache, postural hypotension, tolerance in longer acting preparations
 Beta blockers- used in all patients if no contraindications – titrate resting heart rate to 50-
60/min. Improves survival following MI
 Calcium channel blockers – cause coronary vasodilation and reduce myocardial oxygen
demand by reduction of myocardial contractility. Diltiazem is a commonly used drug.
Dihydropyridine CCBs such as Nifedipine should be avoided as it may cause a tachycardia
and worsen symptoms of angina.

Other medication is also available where first line drugs fail to alleviate symptoms

 Nicorandil – Activates ATP sensitive potassium channels


 Ranolazine – Inhibits the inward sodium channel current in the myocyte. Is contraindicated
in patients with hepatic failure

As your patient may have other comorbid conditions, assess the suitability of the above drugs in
those situations

 Heart failure – avoid diltiazem and verapamil. Amlodipine and ranolazine is safe
 PVD – CCB are favored as beta blockers can cause peripheral vasoconstriction

Risk stratification and further management

 Patients with chronic stable angina should undergo risk stratification. The tools used for risk
stratification are echocardiograms, stress ECGs/Echo and in some cases, myocardial
perfusion scans (not routinely done in Sri Lanka)
 Patients who are symptomatic despite optimum medical therapy, or those with a high risk
score are referred for invasive coronary angiography
Discussion point

This patient, while on therapy presents with acute onset, central chest pain for 4 hours duration. This
pain is associated with intense sweating and several episodes of vomiting.

What is the diagnosis?

The history is suggestive of ACS - Acute coronary syndrome

 This is a medical emergency


 Order a 12 lead ECG immediately. Do not wait for this to manage the patient
 Administer oxygen if required to keep a target SPO2 of 94% - Avoid continuing high flow
oxygen if the patient has adequate saturation
 Give the patient loading doses of aspirin and clopidogrel – Sol. Aspirin 300mg and
Clopidogrel 300mg
 Offer adequate pain relief – Morphine 5mg intravenous (add metoclopramide 10mg IV to
prevent vomiting)
 Send blood for basic investigations including cardiac biomarkers
The ECG shows ST elevations in lead V1 – V6. What will you do now?

STEMI

 Mention that the diagnosis is a STEMI and that you will proceed immediately with
assessment for revascularization
 The options include, primary PCI and thrombolysis.
 In general, revascularization is ideally performed 12 hours from onset of chest pain
 Evidence shows that the revascularization rates and outcomes are better for PCI when
compared with thrombolysis. However, this is when there is ready access to PCI and strict
time-frames are followed.
 It is reasonable to get a cardiology opinion as soon as possible

Primary PCI in STEMI

 If the patient presents to a PCI capable unit, PCI is the therapy of choice and should be
performed within 60 minutes
 If the patient presents to a non - PCI capable unit, and transfer is possible for PCI with < 120
minutes (FMC to device time), transfer should be considered
 Patients who are at high risk, are hemodynamically unstable with LVF or cardiogenic shock
and are at a high bleeding risk do better with PCI and transfer should be considered
irrespective of time
 Patients in whom fibrinolysis is contraindicated should also undergo primary PCI irrespective
of time

Assume that the hospital you are working at does not have PCI facilities

 Mention that your target is to commence thrombolytic therapy within 30 minutes of the
patient arriving in hospital
Thrombolytic therapy

 rtPA (tenecteplase) is available in most


hospitals now. It has better reperfusion rates
and less complications when compared to SK
 This is usually administered as a bolus
injection, with heparin administered before
and after the thrombolytic dose
 Discuss that you will go through all
indications and contraindications for
thrombolysis again. See box
 Administer as described (pharmacology box)
 Monitor the patient while on the
thrombolytic

How would you know that thrombolysis has


worked?

 It is very important to make a quick


assessment of this. If the patient has
achieved adequate reperfusion, chest pain
will diminish, and the ECG will show a more
than 50% decline in the maximum height of
ST elevations within 90 minutes of
completing the thrombolytic dose
How would you monitor the patient?

 Continue monitoring. Start the other medication


 Continue aspirin 75mg and clopidogrel 75mg with high dose statins
 Start the patient on anticoagulation – LMWH, adjust dose in the elderly and in patients with
renal dysfunction
 Unfractionated heparin infusion with a target APTT (1.5 – 2.0) – can also be used
 Start an ACEI and beta blocker if not contraindicated
 Look out for bleeding

What are the options available if thrombolysis fails?

 Call the cardiology unit closest to the hospital


 The best available option in this case is to go for PCI -this is known as rescue PCI
Chest pain and acute coronary syndrome

The ECG shows anterolateral ischaemia and a positive Troponin. How would you manage?

Initial management

 This is considered a non ST elevation acute coronary syndrome. The two types are classified
as UA and NSTEMI, based on positivity of cardiac biomarkers
 Start the patient on loading doses of 300mg aspirin and 300mg of clopidogrel and high dose
statins
 Newer antiplatelet agents including prasugrel and ticagrelor are also available for the
management
 Administer oxygen to maintain saturation above 94%
 Administer IV morphine 5mg for pain relief (with 10mg IV metoclopramide)
 Send for basic investigations and cardiac biomarkers (troponin)
How would you interpret cardiac biomarkers in patients with chest pain?

The patient has been started on the above management. What now?

 The priority now is to decide on the best route of reperfusion. It is best to obtain expert
opinion on this. You may commence anticoagulation if there is a delay in obtaining expert
opinion. UFH and LMWH are the commonly used agents
 Fondaparinux (dose 2.5 mg sc daily) is an alternate agent to use if available
 There is recent interest in offering initial invasive therapy for patients with NSTEMI/UA
 Very high risk patients, which patients who are unstable with cardiogenic shock, left
ventricular dysfunction, life threatening arrhythmias with cardiac arrest, recurrent and
dynamic ST/T changes, especially with transient ST elevations and those ref ractory to
medical management should undergo urgent PCI within 2 hours. Other patients with
intermediate risk factors such as troponin positivity, recent ACS, DM, advanced age and
renal dysfunction should ideally undergo coronary angiogram within 24-72 hours, after
commencement of anticoagulation
The patient is commenced on enoxaparin therapy and is doing well. Anything else to be done?

 Optimize antiplatelet therapy and continue DAPT for a minimum of 1 year


 High dose statins
 ACEI and beta blockers at optimum doses
 Use insulin to maintain a CBS target of 180mg/dl
 Refer to the cardiologist for further follow up

Hypertension

 This is a common associated problem that you will have to address.


 Hypertension will usually be part of a problem in another long case.

In your history and examination, the following key points should be addressed

 Basis for the diagnosis of hypertension and level of hypertension at the point of diagnosis
 Look for features suggestive of a secondary cause for hypertension
Remember that there is no age limit for secondary causes of hypertension. But it is more
likely in patients who are less than 40 years, and have very high blood pressures with acute
target organ damage, or have refractory hypertension
 Look for evidence of target organ damage – retina, cardiac, renal
 Assess other cardiovascular risk factors
 Discuss medication, adverse effects
 Follow up and compliance

How will you evaluate a patient with a secondary cause for hypertension?

Cause Clinical diagnosis Screening investigations


Renal
Renal disease:
AGN Clinical evidence of fluid retention, Renal function tests
uremia, alteration in urine output, UFR and Urine microscopy
CGN hematuria
CKDu Proteinuria
Renovascular disease Origin from endemic areas,
proteinuria
Recurrent history of ‘flash’
pulmonary edema
Worsening of renal functions
following commencement of
ACEI/ATII RB
Renal bruit
Cardiovascular
Coarctation of the aorta Discrepancy in lower extremities 2D echocardiogram
Radio-radial or radiofemoral delay
with variation between brachial
and femoral blood pressures
Ejection systolic murmur between
the scapulae, posteriorly
Endocrine

Conn’s syndrome Proximal muscle weakness as a Serum electrolytes –


manifestation of hypokalemia Hypokalemia is a prominent
feature
Cushingoid phenotype, proximal
muscle weakness, associated
Cushing syndrome hyperglycaemia
CBS, ODST

Phaeochromocytoma Sudden episodic attacks of anxiety,


palpitations and sweating
Thyroid disease (thyrotoxicosis)
Urine metanephrines
Weight loss despite increased
appetite TSH/FT4

Obstructive sleep apnoea Central obesity, daytime Sleep studies


somnolence
ABG for CO2 retention

Other Medication related hypertension –


OCP

Discuss the management principles of hypertension

This should be discussed based on the following themes.

Treatment thresholds and a summary of therapy in hypertensive patients is presented below


Lifestyle modifications

The institution of adequate lifestyle modifications is paramount towards an optimum management


of hypertension. Patient compliance to life style changes and drug therapy is a challenge to the
clinician and therefore patient education and motivation is important.

Cessation of smoking

Moderation of alcohol consumption

Diet
A diet plan with local and cultural acceptance should be formulated with the principles stated below
in consultation with a dietician where necessary. The DASH diet plan outlines a diet rich in fruits and
vegetables; high in low-fat dairy products, potassium, magnesium, and calcium; and low in total
saturated fat

Salt consumption
A daily intake of 5– 6 g of salt is recommended. (1 teaspoon salt = 5 grams)

BMI
Maintenance of a healthy body weight (BMI of about 23 kg/m2) and waist circumference (<80cm in
females and <90cm in males) is recommended.

Physical exercise
Hypertensive patients, especially those who follow a sedentary life style should be advised to
participate in at least 30 min of moderate-intensity dynamic aerobic exercise (walking, jogging,
cycling or swimming) on 5–7 days per week

Pharmacological management – antihypertensive therapy

There are several classes of antihypertensive drugs available. The first line medication for the
management of hypertension includes the following drug classes

1. Renin – angiotensin system blockers – ACEI and angiotensin II receptor blockers


2. Calcium channel blockers
3. Diuretics – thiazides

Selection of these antihypertensive medication is based on compelling indications, compelling


contraindications and other factors such as availability. The important characteristics of the first line
antihypertensives are summarized below. It is important in practice to assess the response to
antihypertensive therapy. Combinations of antihypertensive drugs may be considered in patients
with poor response to monotherapy.

Preferred combinations
 Thiazides and ARB/ACEI
 Thiazides and CCB
 ACEI/ARB and CCB
 Combination of ACEI and ARB is not recommended for the management of hypertension
Diuretics ACE inhibitors ARB CCB
Drugs HCT, Indapamide Captopril, Losartan, Dihydropyridine –
Enalapril, Telmisartan Nifedipine,
Lisinopril Valsartan Amlodipine
Non DHP –
Verapamil,
Diltiazem
Mechanism of Blocks the Na/Cl Inhibits the RAS Blocks AT II Block voltage gated
action co-transport by blocking receptors calcium channels in
mechanism in the conversion of AT I vascular smooth
DCT to AT 11 muscle and
myocytes
Compelling Isolated systolic Heart failure Intolerant to ACEI LVH
indications hypertension in Reduced LV Heart failure Atherosclerosis
the elderly function after MI Nephropathy in
Diabetes (type 1 association with
and 2) type II DM
Diabetic renal
disease
Non diabetic
renal disease
Adverse effects Metabolic Cough, Hyperkalemia, Dihydropyridine
disturbances, Hyperkalemia, Angioedema CCB
Hypokalemia, Angioedema, (Rare) Headaches, flushing,
hyponatremia, Leucopenia, ankle edema,
hyperuricaemia, Anaplasia cutis in worsening of heart
Increased TG, the fetus, failure, gum
insulin resistance, Cholestatic hyperplasia
Impotence jaundice Non
Dihydropyridine
CCB
Bradycardia, heart
block, constipation
(> verapamil)
Contraindications Gout Pregnancy, Pregnancy, DHP - Heart failure,
Bilateral renal Bilateral renal Non DHP- Heart
artery stenosis, artery stenosis, block and
Hyperkalemia Hyperkalemia bradycardia
Notes Efficacy drops Monitor serum May be less Effective
when the GFR < 50 creatinine and effective than antihypertensives
electrolytes CCB DHP should not be
used as montherapy
in proteinuria

Second line antihypertensives may be considered in patients with di fficult to control and resistant
hypertension.

Resistant hypertension is defined as, Hypertension is defined as resistant when a therapeutic


strategy that includes appropriate lifestyle measures plus a diuretic and two other antihypertensive
drugs belonging to different classes at adequate doses fails to lower SBP and DBP values to less than
140 and 90 mmHg (150 and 90 mmHg in people ≥60 years), respectively.
These agents are,

1. Beta blockers
2. Alpha blockers
3. Direct vasodilators
4. Centrally acting drugs
5. Aldosterone antagonist – spironolactone

The important pharmacological characteristics of these medications are summarized below

Beta blockers Alpha blockers Centrally Vasodilators Aldosterone


acting antagonists

Drugs Atenolol Prazocin Methyldopa Hydralazine Spiranolactone

Mechanism of Blocks beta Blocks alpha Central Vasodilation Blockade of


action receptors (b1 in adrenergic receptors blockade of the
the heart) in vascular smooth sympathetic aldosterone
muscle output receptor
Reduces renin
release (b1) and
NE release (b2)

Some have alpha


blocking effect
(Labetolol,
carvedilol)

Compelling Post myocardial Phaeochromocytom Hypertension Severe PIH Associated


indications infarction a in pregnancy heart failure
Obstructive
uropathy
Adverse effects Heart block, Orthostatic Dry mouth, Fluid Hyperkalemia
Worsening of hypotension, 1 st Lethargy, retention, Gynaecomasti
cardiac failure, dose hypotension, Hemolytic Drug induced a
Bronchospasm, Ankle edema, Drug anaemia Lupus, Reflex
Depression, tolerance, Reflex Orthostatic tachycardia
Nightmares. PVD, tachycardia and hypotension
Impaired glucose worsening of heart
tolerance failure

CI Asthma, Heart Orthostatic Orthostatic Hyperkalemia


block, Depression hypotension, Heart hypotension
failure

Treatment targets

This is in fact one of the most controversial topics in hypertension. Guidelines differ on their targets.
A reasonable target is, 140/90 in low/moderate risk hypertensives and 130/80 mmHg in high-risk
hypertensives (with diabetes, cerebrovascular, CV, or renal disease)

Other aspects of management

 Statins and antiplatelet drugs are considered in patients with existing cardiovascular disease
and in those with high cardiovascular risk
Chronic cough and hemoptysis

Introduction to the patient

Background – highlight the relevant aspects of the medical background of the patient

Presenting complaint

Chronic cough is cough lasting for more than 8 weeks

Description of the presenting complaint

 A comprehensive discussion on the symptom is important. This should include a


chronological description
 Describe the onset, evolution and progression of the symptom
 Describe the nature of the cough as productive or non-productive. If the cough is productive
in nature describe the characteristics of the sputum
 Describe the frequency of cough and diurnal variation of symptoms
 Describe the exacerbating and relieving factors of the cough
 Describe the associated symptoms

Analysis of the presenting complaint

Ask specific questions to identify the possible cause. Recall that chronic cough can be due to
respiratory disease, cardiac failure, drugs and gastro-esophageal reflux disease

Specific questions
Cardiac
Left ventricular failure Cough in patients with left ventricular failure is predominantly
nocturnal. Ask for associated exertional shortness of breath,
orthopnea and paroxysmal nocturnal dyspnea and ankle
edema
The patient may also have a suggestive aetiological factor for
cardiac failure
Respiratory disease
Upper airway syndromes Ask for associated upper airway symptoms such as rhinorrhea,
causing chronic cough (post nasal discharge and nasal congestion. The patient also has
nasal drip) significant throat irritation and features of recurrent sinusitis

The cough is classically of a diurnal variation and may have


Bronchial asthma and cough certain exacerbating factors that the patient will identify
variant asthma
There is no diurnal variation of cough in these patients and
COPD symptoms are persistent throughout the day
Ask for progressive shortness of breath with acute
exacerbations
COPD patients usually have a baseline level of shortness of
breath between exacerbations
Acute exacerbations are highlighted by cough with productive
sputum
A significant history of smoking should also be extracted
Hypersensitivity pneumonitis Take a careful history of occupational and environmental
exposures

Bronchiectasis The cough will be characteristically associated with copious


amounts of sputum formation. The sputum production has
postural alteration
Ask for childhood history of recurrent respiratory tract
infections, past TB

Bronchial carcinoma The patient will have recent onset cough with hemoptysis
Also ask for alarm symptoms such as loss of appetite, loss of
weight and other features suggestive of local spread such as
drooping of the eyelids, hoarseness of voice, swelling of the
face
Also ask for evidence of paraneoplastic syndromes such as
wrist pain, ataxia, muscle weakness and pigmentation

A significant history of smoking is also relevant


Tuberculosis
These patients will have associated constitutional symptoms
such as loss of appetite and loss of weight
Also ask for evidence of low grade fever with night sweats
Ask for an epidemiological history suggesting close contact or a
past history of TB
Interstitial lung disease
These patient will have progressive shortness of breath in
association with dry cough
Ask for associated connective tissue disease and occupational
exposures
Gastro-esophageal reflux Ask for associated dyspeptic symptoms and abdominal pain
disease The cough in these patients is also worse at night
Drugs ACEI, NSAIDs and beta blockers

 After you have done this, establish the patient’s course through healthcare services,
including the investigations and interventions which have been performed
 This may give you a clue to the diagnosis, but do not be biased in your evaluation based on
this
 Evaluate the current functional state of the patient

Past medical history

 Describe as relevant

Family history

 Atopy and bronchial asthma


Social history

 Discuss the burden of smoking


 Give a detailed description of the occupational and environmental exposures as emphasized
above
 Contact to animals and pets
 Describe the impact of the symptoms on day to day functioning
Examination

General examination

 Look for Horner syndrome and facial puffiness with distended neck veins (SVC obstruction)
 Plethora – chronic hypoxia
 Facial rashes
 Lymphadenopathy – supraclavicular nodes in patients with bronchial carcinoma
 Examine the hands for clubbing (bronchiectasis, ILD, bronchial carcinoma), nicotine stains,
look for evidence of connective tissue disease (rheumatoid hands, systemic sclerosis)
 Ankle edema – cardiac failure
 Lower limb rashes suggestive of eczema

Cardiovascular

 Look for evidence of left ventricular failure

Abdomen

 Epigastric tenderness and abdominal distension

Nervous system

 Perform a quick screening examination

Hemoptysis

Introduction to the patient

Background – highlight the relevant aspects of the medical background of the patient

The age of the patient is important as it modifies your differential diagnosis

Presenting complaint

The presenting complaint will be passage of blood-stained sputum

Description of the presenting complaint

 Make sure that you do an extensive description of the prese nting complaint in chronological
sequence
 Make sure you provide enough information to differentiate hemoptysis from hematemesis
 Describe the onset, evolution and progression of the symptom
 Describe the quantity and color of the hemoptysis – fresh or altered blood
 Ask about possible exacerbating and relieving factors
 Ask about the associated symptoms
Analysis of the presenting complaint

Recall that hemoptysis can be due to,

 Respiratory disease
 Cardiac disease
 Bleeding disorders

Specific questions
Cardiac disease
Pulmonary edema Ask for associated nocturnal cough, production of pink
frothy sputum, progressive exertional shortness of
breath, orthopnea and paroxysmal nocturnal dyspnea
Ask for past history of rheumatic fever and valvular
heart disease
Look for other possible causative agents for left
ventricular failure
Respiratory disease
Pneumonia Ask for fever, pleuritic chest pain and hemoptysis with
predominantly rusty colored sputum production

TB These patients will have associated constitutional


symptoms such as loss of appetite and loss of weight
Also ask for evidence of low grade fever with night
sweats
Ask for an epidemiological history suggesting close
contact or a past history of TB

Aspergillosis Recent onset hemoptysis in a patient with a past


history of TB may point towards this diagnosis

Bronchial carcinoma The patient will have recent onset cough with
hemoptysis
Also ask for alarm symptoms such as loss of appetite,
loss of weight and other features suggestive of local
spread such as drooping of the eyelids, hoarseness of
voice, swelling of the face
Also ask for evidence of paraneoplastic syndromes such
as wrist pain, ataxia, muscle weakness and
pigmentation

Bronchiectasis A significant history of smoking is also relevant

The patient will have associated cough. The cough will


be characteristically associated with copious amounts
of sputum formation. The sputum production has
postural alteration
Ask for childhood history of recurrent respiratory tract
Pulmonary vasculitis and pulmonary infections, past TB
renal syndromes
Rare causes of hemoptysis
Pulmonary embolism
Other bleeding disorders These are suggested by bleeding episodes involving
other sites

 After you have done this, establish the patient’s course through healthcare services,
including the investigations and interventions which have been performed
 This may give you a clue to the diagnosis, but do not be biased in your evaluation based on
this
 Evaluate the current functional state of the patient

Past medical history

Drug history – use of anticoagulant/antiplatelet medication

Social history – explore the patient’s concerns

Examination

General examination

 Look for Horner syndrome and facial puffiness with distended neck veins (SVC obstruction)
 Plethora – chronic hypoxia
 Lymphadenopathy – supraclavicular nodes in patients with bronchial carcinoma
 Examine the hands for clubbing (bronchiectasis, bronchial carcinoma), nicotine stains
 Ankle edema – cardiac failure
 Look at the skin for evidence of petichiae, purpura and ecchymoses

Cardiovascular

 Examine for evidence of MS


 Left ventricular failure

Respiratory

 Detailed examination for the above

Case summary

Mrs. X, a 45-year-old patient with type 2 diabetes for 8 years presented with a history of chronic
cough for 3 months. She felt unwell throughout this period and noted a mild fever towards the
afternoon. The cough was dry at times but was productive on many occasions. Over the last few days
she had noted some blood in the sputum. She does not give any contact history with tuberculosis and
neither does she give a past history of tuberculosis. Apart from diabetes there are no other co
morbidities. She had lost 8 kg over the last 3 months

On examination she appeared wasted. There was no lymphadenopathy and the BCG scar was
present. There was no clubbing. The cardiovascular system, nervous system and abdomen were
clinically normal on examination. On examination of the respiratory system, the trachea was
deviated to right and the movements were less on the right apical area. This area was dull to
percussion and had bronchial breathing on auscultation.
What is the most likely diagnosis?

Pulmonary TB

Why do you say so?

This patient has presented with a 3-month history of cough and a recent onset hemoptysis. She also
complains of ill health and loss of weight and evening pyrexia.

1. TB is high on the list even though there is no contact history or past history of TB.
2. Bronchial carcinoma can also present as above but evening pyrexia is unusual and more
suggestive of a chronic infective process
3. Bronchiectasis may also be considered in the differential diagnosis due to the history of
productive cough

The examination reveals features are suggestive of right apical fibrosis. This is evidence of past TB.

Therefore, the clinical history with apical fibrosis as evidence of past TB makes a diagnosis of TB a
very strong possibility.

What are the investigations you would like to perform in this patient?

The initial investigations should be performed with the objective of narrowing down the differential
diagnosis.

 FBC
 Inflammatory markers – ESR and CRP
 Sputum for AFB and culture – 3 early morning expectorated samples of sputum
 CXR
TB – Shows upper lobar infiltrates with/without cavitation
Bronchial malignancy – pulmonary nodules (solitary or multiple)
Bronchiectasis – Ring shadows and tramline opacities
 Tuberculin skin test – Mantoux test
A positive test only indicates past infection with Mycobacterium TB or other non
tuberculous mycobacteria
Is also positive in individuals who have had BCG vaccination
However, a strongly positive (induration >15mm) test is highly suggestive of TB
Furthermore, a tuberculin skin test may be falsely negative in patients who a malnourished
or immunocompromised

Discuss the management of this patient

The main objectives of treatment in a patient with TB are

 Curing the patient of TB


 Prevention of early and late complications
 Prevention of relapses
 Prevent the transmission of TB in the community
 To prevent the emergence of drug resistant TB
Discuss the following options in the management

 Educate the patient regarding the disease and treatment


 Adopt measures to stop the spread of TB within the wards – face mask, cough etiquette,
proper disposal of sputum
 Anti TB drugs
Isoniazid, Rifampicin, Ethambutol, Pyrizinamide and Streptomycin

Mechanism of Adverse effects


action
Isoniazid Bactericidal Hepatotoxicity, peripheral neuropathy (co administration of
pyridoxine reduces the risk), hypersensitivity reactions
Rifampicin Bactericidal Hepatotoxicity (cholestatic), discolouration of body fluids,
hematological (anaemia and thrombocytopenia), hypersensitivity
reactions
Drug interactions
Pyrizinamide Sterilizing Hepatotoxicity, hyperuricaemia and gout, GI side effects
Ethambutol Bacteriostatic Optic neuritis
Streptomycin Bactericidal Ototoxicity, renal impairment

 The treatment of TB with these drugs consists of 2 phases. These are the intensive phase
and the continuation phase

 The exact regimens used are given below

This patient belongs to category 1

Category 1 treatment

 Category 1 treatment is given to all new patients


Smear positive and negative pulmonary TB and extra pulmonary TB
 The intensive phase is commenced with HRZE daily for 2 months
 After the intensive phase is completed a sputum smear examination should be performed. If
this is negative the continuation phase is commenced
 If it is positive the intensive phase should be continued for 1 more month. At the end of this
period a repeat sputum smear is done. If this is persistently positive drugs should be stopped
for 3 days and sputum should be taken for ABST and culture
 Following this the continuation phase is commenced
 In the continuation phase HR is given for 4 months
 Sputum smear examination is done at the end of 5 months and 6 months
 If sputum is positive at 5 months this is regarded as a treatment failure and category 2
treatment should be commenced
 If sputum is negative at the end of 6 months treatment may be discontinued
Other aspects of management

 Perform baseline liver function tests before commencement of treatment


 Notification
 Referral of patients for DOTS
 Follow up the patient
Swelling of the body

Introduction to the patient

Background – highlight the relevant aspects of the medical background of the patient

Presenting complaint

The presenting complaint is swelling of the body

Description of the presenting complaint

 Describe the chronological presentation of the symptom


 This should include the onset, evolution and progression of the body swelling
 Ask for aggravating factors, relieving factors and associated factors
 In your description it is important to describe the profile of edema and the distribution. The
distribution of edema can be either localized or generalized
 In patients with generalized edema note the presence of a disproportionate abdominal
distension (ascites)
 In a long case the focus is usually on generalized body swelling

Analysis of the presenting complaint

 Recall that generalized body swelling can be due to,


 Cardiac failure/right ventricular failure
 Respiratory disease with cor-pulmonale
 Renal disease – this includes a spectrum of conditions – nephrotic syndrome, nephritic
syndrome and CKD
 Liver disease – cirrhosis with portal hypertension
 Gastrointestinal – malabsorption syndromes and protein loosing enteropathies
 Hypothyroidism
 Drug induced

Specific questions

Cardiac failure The pattern of edema in patients with cardiac failure is


predominant dependent edema which is worse towards the end
of the day and with exertion
Ask for associated exertional shortness of breath with
orthopnea and paroxysmal nocturnal dyspnea
Also look for a suggestive aetiological factor for cardiac failure,
especially ischaemic heart disease and valvular heart disease

Right ventricular failure Ask for past history of respiratory disease (COPD/ILD)
Renal disease
Nephrotic syndrome The edema in nephrotic syndrome has predilection to the face
and periorbital region. The edema is of gradual onset and is worse
in the morning. These patients can develop gross edema
This should be established in the chronological presentation
Ask also for associated urinary symptoms- excessive foamy urine
The patient may also be aware of excessive protein in the urine
If the features are suggestive of nephrotic syndrome, proceed
with questions on the possible aetiology

Diabetes mellitus
History of infective disease or associated risk factors for retroviral
infection and Hep B/C
History suggestive of autoimmune disease – joint pain, alopecia,
facial rashes, rheumatoid arthritis, Sjogren syndrome
History suggestive of hematological malignancy
Lymphoma – fever, night sweats, myeloma – recent onset sinister
backache
History of solid organ malignancy – breast, bronchial
Loss of appetite, peripheral neuropathy, autonomic neuropathy –
amyloidosis
Look for possible drugs causing NS

Also ask for complications, such as thrombotic events, infections


Nephritic syndrome The pattern of edema is similar to that of nephritic syndrome, but
is of quicker onset and progression
Ask the patient for associated urinary symptoms – declining urine
output and hematuria

If the features are suggestive of nephritic syndrome, proceed


with questions on the possible aetiology

Recent skin sepsis, sore throat (PSGS)


Features of autoimmune disease/ vasculitis – skin rashes, nervous
system involvement
Chronic kidney disease The pattern of edema is similar to the above
Ask for symptoms suggestive of uremia – decreased appetite,
nausea, vomiting, metallic taste in the mouth, hiccups,
myoclonus, generalized pruritus
Associated episodic pulmonary edema
Liver disease The patient will have prominent abdominal swelling
Ask for complications of decompensated liver disease –
hematemesis/malaena, features of hepatic encephalopathy,
yellowish discoloration of the eyes
Malabsorption and protein Ask for chronic diarrhea suggestive of small bowel origin
loosing syndromes
Hypothyroidism Lethargy, increased somnolence
Drugs A detailed drug history is essential

 After you have done this, establish the patient’s course through healthcare services,
including the investigations and interventions which have been performed
 This may give you a clue to the diagnosis, but do not be biased in your evaluation based on
this
 Evaluate the current functional state of the patient

Drug history

 CCB, steroids, NSAIDs, thiozolidinediones


Past medical and surgical history

Social history

 Discuss on alcohol use and smoking


 Discuss the impact of these symptoms on the patient’s life

Examination

General examination

 Pallor,
 Icterus – may suggest cirrhosis as the cause
 Plethora – associated with chronic hypoxia in respiratory disease
 Clubbing
 Look for peripheral stigmata of cirrhosis
 Lymphadenopathy
 Look for skin changes associated with CKD
 Edema
Cardiovascular examination

 Look for elevated JVP – congestive cardiac failure, cor-pulmonale


 Look for features of cardiac failure such as dilated apex, functional murmurs (MR and TR),
added sounds (S3)
 Examine for a loud P2 and parasternal heave – pulmonary hypertension

Respiratory

 Examine for pleural effusions (nephrotic syndrome)


Abdomen

 Look for Organomegaly – splenomegaly – liver disease

Case discussion

Summary

A 43 year female who is a diagnosed patient with Hypertension and Hyperlipidaemia for 3 years
duration presented with generalized edema for 1 week duration. These symptoms were associated
with frothy urine, frequency and nocturia for 1 month duration. She also complains of symmetrical
inflammatory type arthritis affecting her metacarpophalangeal joints. She has poor compliance to
medication, diet and lifestyle modifications recommended for hypertension. Examination is significant
for grade II hypertensive retinopathy, generalized edema and ascites

What is your diagnosis?


Nephrotic syndrome
Why do you say so?
Patient presented with gross oedema which was first noted as a periorbital swelling. There is also a
history of frothy urine which has been confirmed by urine ward test
Absence of haematuria or elevated blood pressure shows that the chance of associated nephiritic
syndrome is less likely
Nephrotic syndrome

How would you confirm a diagnosis of nephrotic syndrome?

This should follow the definition of nephrotic syndrome

 Perform a UFR to demonstrate the presence of proteinuria and proceed with quantification
– 24 urine protein - >3.5g/24h or uPCR > 2000 mg/g ( >200 mg/mmol), or > 300 mg/dl
 Review the liver function tests to assess the serum albumin - < 2.5 g/dl
 Perform a serum cholesterol - > 200mg/dl

How would you proceed?

 The next important step is to assess the aetiology. This can be idiopathic or secondary.
Discuss important points in the history and examination which may be suggestive of a
secondary cause. If this is the case, proceed with specific investigations

DM This is by far the commonest cause, but FBS


think of an alternate aetiology in patients
with DM who present with NS early in the
illness, have atypical features (mixed
picture, accelerated renal dysfunction)
and absence of proliferative retinopathy
Infective disease Hep B/C, HIV Hep B/C serology
Look for concomitant liver disease HIV
High risk sexual behavior, blood
transfusion
Autoimmune SLE is another important cause to think ANA, C3/C4
of. Review the classical features in the
history and examination
Neoplasms Lymphoma is an important cause to USS/CT abdomen
consider
Look for the classical features of fever,
loss of appetite and weight,
lymphadenopathy
Other malignancies such as carcinoma of
the bronchus can also present with NS ESR
Myeloma Serum protein and urine
protein electrophoresis
Drug induced This is rare

A renal biopsy will be required in almost all cases of adult onset NS. Discuss the importance of this in
deciding your management. The histological pattern will be important in deciding the management
What are the principles of management?

Definitions

Relapse Recurrence of nephrotic range proteinuria after


complete remission for > 1month
Frequent relapse 2 or more relapses within 6 months
Remission Reduction of proteinuria to < 0.2 g/d and serum
albumin > 3.5g/dl
Steroid resistant Persistence of proteinuria despite 1mg/kg
steroids for > 16 weeks
Steroid dependent 2 consecutive relapses during steroid therapy
or within 14 days of completing therapy

 Symptomatic management should be commenced. Start a daily weight chart and input
output chart
 General – restrict salt in diet. Prescribe a normal protein diet
 Use diuretics for management of edema – frusemide is the most commonly used drug.
Combinations of frusemide and metalazone are also useful
 Discuss that you would measure the weight of the patient and target a daily weight loss of
0.5-1kg
 Co-administration of albumin increases the efficacy of frusemide. If albumin is not available
cryo poor plasma may be used. (FFP is better avoided as it can precipitate thrombosis in an
already procoagulant state)
Albumin acts as a carrier molecule for frusemide at the renal tubular level and expands the
intravascular volume
Dose – 1g/kg – twice a day
 Start the patient on anti-proteinuric medication – ACEI or ATII RB
 Statin therapy is controversial and may be considered in patients with cardiovascular risk
factors
 Administer prophylactic LMWH in patients with albumin levels below 2.0g/dl

Consider definitive management

 This depends on the underlying cause and the renal histology.


 This is complicated and is knowledge of this is not expected from a medical student
 Options include – Steroids and other immunosuppressive medication – such as CP, CyA,
MMF

The consultant decides on a renal biopsy for this patient. As a house officer, how would you
prepare this patient for renal biopsy?

 Renal biopsy preparation – informed written consent is essential


 The patient needs FBC, coagulation profile and grouping and DT before the procedure
 Following the procedure monitor the patient’s vital parameters for any evidence of bleeding.
Ask the patient to collect urine to look for hematuria. Hematuria, a few hours after the
procedure is acceptable if this clears over the next few hours
 The patient should ideally be placed in a bed in the acute cubicle of the ward
 Call for immediate senior opinion if the patient shows any signs of internal bleeding
What are possible complications of nephrotic syndrome?
Complications can be disease related and treatment related

Disease related
Pleural effusion and pulmonary oedema
Immune suppression and recurrent infection (loss of immunoglobulins)
Hypercoagulable state

Drug related
Side effects of steroid
Steroids

What is the commonest cause for nephrotic syndrome in adults?


Membranous glomerulonephritis

Nephritic syndrome

24 old manual worker from Negombo, presenting with passage of red colored urine for 4 daysduration.
Passage of red colored urine is throughout the stream and it is not associated with other urinary
symptoms such as hesitancy, poor stream or dysuria. He has noticed a mild reduction in his urine
output. Over the last few days he has noticed swelling around his eyes which was more prominent in
the morning. He is free of any significant past medical or surgical history except for itchy skin rash
which has been troubling him for last few weeks with smelly discharge. He has consulted a general
practitioner and was found to have elevated blood pressure and was advised to get admitted.

On examination mild peri orbital oedema was present. There was a healing skin lesion over left foot.
His blood pressure was recorded as 160/100mmhg. Lung fields were clear. Urine ward test was 2+ for
proteins.

What is your diagnosis?

Acute nephritic syndrome

Why do you say so?


This patient presented with haematuria associated with mild peri orbital oedema and decreasedurine
output. Furthermore, his blood pressure has been elevated and urine ward test shows proteinuria.
These features are keeping with a presentation of acute nephritic syndrome

How are you going to investigate this patient?


 Confirm haematuria
 UFR – look for RBC cells and cast (look for pus cells to check for evidence of UTI)
 Exclude UTI, associated nephrotic syndrome
 Urine culture, Serum albumin
 Identify aetiology
Further clinical assessment should be performed with the intention of looking for a seco ndary cause
and to assess the severity of the condition

Condition History and examination Investigations


Post streptococcal Preceding skin or throat Anti DNAase B, ASOT,
glomerulonephritis infection complement levels
Lupus nephritis Skin rashes, oral ulcers, ANA, anti dsDNA, complement
alopecia levels, renal biopsy
and other features of SLE
IgA nephropathy Recurrent episodes of Renal biopsy
haematuria immediately
following RTI
Small vessel vasculitis Features of systemic illness, ANCA, renal biopsy
pulmonary manifestations
Membranoproliferative GN Multiple causes Renal biopsy to confirm
Investigate for causes
(i.e. Hepatitis C, malignancy,
vasculitis)
Mesangioproliferative GN Multiple causes Investigate for the cause

 As mentioned above investigations such as renal function tests and serum electrolytesshould
be performed to assess the severity of the condition

How will you manage this patient?

 Admit the patient


 Manage symptomatically
 Maintain input output chart, BP measurements
 Monitor for complications and manage as necessary – hypertension
 Specific treatment required in selected cases – i.e. SL

Chronic kidney disease – history and examination

Presenting complaint

 The patient will usually present with complications of ESRF or for a regular session of
hemodialysis
 Make sure that you describe the presenting complaint in detail before progressing to the
past medical history

Discuss the past medical history

 Describe how the diagnosis was established and initial therapy initiated
Discuss the possible aetiology in the history

 Common themes to discuss


 Inherited renal disease and structural abnormalities – ask for a family history of renal
disease, childhood history of renal disease and recurrent urinary tract infections in childhood
 Glomerular disease – ask for a childhood history of glomerular disease or recent glomerular
disease. The patient may know the results of renal biopsy etc.
 Autoimmune disease – Past history of SLE
 DM
 HT
 Chronic tubulo-interstitial nephritis - Ask for a possibility of CKDu – obtain an
epidemiological history of CKDu and a detailed history of residence, occupational exposures
 Get a detailed history on drug exposures – i.e. NSAID use

Describe the complications the patient has developed

 Features of uremia and fluid overload


 Cardiovascular complications
 Anaemia and contributing causes should be discussed in detail
 Bone disease
 Episodes of gout

Discuss the current and future management plans for the patient

 Describe the current use of renal replacement therapy in the management of the patient
 If the patient is on HD, describe – the mode of vascular access, complications of HD and
frequency of HD. Describe the adequacy of HD and symptoms between episodes of HD
 Describe the current state of plans for transplantation and availability of the donor
Discuss the psychosocial issues of CKD in detail

 This can include problems related to financial constraints, psychological issues

Examination

General examination

 Pallor
 BMI and body weight
 Skin manifestations of CKD – hyperpigmentation, sclerosis, scratch marks, calciphyalxis – skin
necrosis due to calcium deposition in patients with severe hypercalcaemia
 Ankle edema
Cardiovascular system

 JVP
 Pericardial effusion and pericardial rubs
 Features of pulmonary edema
 Cardiomegaly – uremic cardiomyopathy
 Examine the vascular access sites
 AV fistulae for functional status
Respiratory system

 Pleural effusions
 Features of pulmonary edema
Abdomen

 Free fluid
 Ballottable masses suggestive of PCKD

Neurology

 Asterexis
 Features suggestive of peripheral neuropathy

Case discussions – Chronic kidney disease

This is a very common long case theme. Therefore, you should have a good idea on the management
problems in the patient with CKD

General idea on the management

The management plan in a patient with CKD depends on the stage of the disease

How would you stage a patient with chronic kidney disease?

The current classification of CKD


incorporates the GFR (calculated
from the serum creatinine using
equations such as the MDRD and
CKD – EPI formula) and albuminuria
(Gradation of color from green to
red signifies increased risk and
worsening prognosis)
What are the management priorities in a patient with early stage CKD?

This question can be asked in practically any long case featuring a metabolic theme. (I.e. DM, HT)

Preventing the progression of the disease

 Reduction of intraglomerular pressure and proteinuria is paramount in this aspect. ACEI and
ATII are first line medication. If these medications are contraindicated or not tolerated
alternate medication may be used. CCBs such as diltiazem and verapamil also have
antiproteinuric and renal protective effects. Adequate glycaemic control and optimizing the
management of hypertension is also important at preventing progressive renal dysfunction
in patients with diabetic nephropathy

What are your blood pressure control targets?

 Blood pressure control targets in CKD are controversial, but a target of < 130/80 is
recommended in current guidelines in patients with CKD and DM or significant proteinuria (>
70 mg/mmol), and < 140/90 in others

Managing other co morbidities and cardiovascular risk factors

 This is also important in the management of CKD as cardiovascular causes are a leading
cause of morbidity and mortality in this context
 Advise the patient on weight reduction, control of hyperlipidaemia, smoking cessation and
glycaemic control. In patients with DM remember that intensive gycaemic control can cause
frequent hypoglycaemias, especially in patients with impaired renal function and gl ycaemic
targets should be set on a case to case basis

Case summary
A 56 year male presented with bilateral leg swelling, severe loss of appetite, nausea, vomiting and
pruritus. He is a diagnosed patient with diabetes for 20 years and a hypertensive for 10 years. He had
not been on regular medication or follow up. Since of late he has noted facial puffiness in the
morning as well. The urine output has reduced progressively over the last few months and he feels
shortness of breath on walking about 500m. He had seen a general practitioner for these complaints
and at the initial assessment was found to have high blood pressure and low haemoglobin. He was
also told that his kidneys may be diseased and needs further evaluation. Apart from diabetes and
hypertension he does not have any other co morbidities and does not give a past history of renal,
liver or cardiac disease. He worked as a superintendent in a tea state and had a good income. He has
good family support and a brother who is willing to donate a kidney if necessary

On examination the patient was oedematous and pale. There were scratch marks on his arms and
thighs and the skin was dry and scaly. There was bilateral pitting oedema as well as facial puffiness.
The blood pressure was 180/100 mmHg with no difference between the arms. An ejection systolic
murmur was audible in the aortic region which was not radiating to the neck. There were bilateral
fine crepitations of lung bases. The abdominal examination did not reveal any organomegaly.
Nervous system was clinically normal.
The patient with ESRF

This is a very common long case management problem

Outline your management plan for this patient

 The optimum management plan for a patient with ESRF should focus on three basic aspects
 These include,
Preparation and decision on the mode of renal replacement therapy
Management of complications
Management of symptoms of uremia

General management issues

 Patient education and counselling – it is important to educate the patient on the disease and
counsel them on the available options and management strategies
 Pulmonary edema – diuretic therapy
 Cardiovascular risk management
Manage hyperlipidaemia
Blood pressure management
Hyperkalemia is frequently a problem in patients with ESRF, and hypertension may have to
be managed with CCB and alpha blockers
Management of anaemia, hyperphosphatemia and hypercalcemia is also important for
cardiovascular risk protection
 Diet
Protein restriction – 0.8g protein/ideal body weight/day (0.6 in severe uremia) with
adequate calories (35kcal/kg/d). The protein content in a Sri Lankan diet is mainly derived
from plant proteins (pulses). Plant sources have a low protein content. And strict restriction
is not often required. Restricted potassium and phosphate should be considered. High
potassium containing foods include fruits such as banana, mango and papaya and some
green leafy vegetables. Cutting, soaking and boiling vegetables reduce the potassium
content. Phosphate content is high in dairy products.
 Salt consumption should also be restricted in patients with significant edema
 Symptom control of uremia
Restless legs – clonazepam, gabapentin
Pruritus – emollients, antihistamines
Nausea and vomiting – metaclopramide

Renal replacement therapy

 The options available are long term hemodialysis, continuous ambulatory peritoneal dialysis
and renal transplantation. Selection of patients for these options are done by specialist
nephrologists and patients should be referred when necessary
 Hemodialysis and renal transplantation are the usual modes of renal replacement therapy
practiced in Sri Lanka
 Renal transplantation – this is performed only at specialized centers. Both live donor and
cadaveric transplants are offered
 Hemodialysis is offered in both the government and private sectors. In the government
sector preference is given to patients in the transplant program. The HD prescription should
be decided by the nephrologist with the ideal prescription tailored based on individual
needs. The patient should be referred to the vascular surgeon to optimize the route of
vascular access
 HD must be performed with utmost care in patients with pre-existing cardiac dysfunction
 Continuous ambulatory peritoneal dialysis (CAPD) is also an emerging modality for patients
in Sri Lanka

How would you monitor a patient on hemodialysis?

 It is most important to closely monitor the cardiovascular status with pulse rate, blood
pressure and continuous cardiac monitoring with ECG
 It is important to discuss the common complications in a patient on HD
 Acute complications
Vascular access related bleeding
Thrombotic complications of the AVF and circuit
Dialysis disequilibrium syndrome – this is seen with patients with severe uremia, when
aggressive dialysis is initiated, Cerebral edema occurs due to rapid reductions in serum
osmolarity and the patient may develop seizures
If this occurs initiate immediate therapy with mannitol or hypertonic saline
Cardiac arrhythmias
Dialyzer reactions
 The patient also has risk of chronic complications such as cardiovascular disease and dialysis
related amyloidosis

How would you manage this patient’s anaemia?

Recall the pathophysiology

 The patient requires a comprehensive assessment as to the cause for anaemia. The main
factor contributing to anaemia is a relative deficiency of EPO. This is now recognized to be
due to abnormal hypoxia sensing mechanisms that promote EPO synthesis. Apart from this
there are many other factors which contribute to anaemia
 Dietary deficiency of iron, folate and B12
 Chronic inflammatory state and anaemia of chronic disease
 Marrow disease due to fibrosis – precipitated by hyperparathyroidism
 Uremic marrow failure and reduced red cell survival due to uremia
 Bleeding – stress ulceration and platelet dysfunction

Therefore,

 Mention that you will look at the full blood count and review the status of anaemia. Look at
the red cell indices
 Request iron studies, specifically serum ferritin and transferrin saturation. Serum ferritin <
500 and transferrin saturation less than 30% indicates requirement for iron replacement
therapy
 Look for clinical sites of bleeding and evaluate for further investigations
 Iron replacement may be an issue in patients with CKD due to problems of absorption. IV
iron is a useful method of iron replacement in patients with CKD especially if they are on HD
 Calculation of the dose of intravenous iron is based on the weight, age and target
hemoglobin concentration
ESA therapy

 EPO is the most commonly used agent. This should be started at a dose of 200IU, 2-3 times
weekly and gradually increased. The route of administration is usually subcutaneous
 Monitor the patient for complications of EPO therapy. Blood pressure monitoring is
important

What would you think of if Hb levels do not rise despite EPO therapy?

 Make sure that iron deficiency anaemia is adequately corrected


 Pure red cell aplasia is a rare complication of EPO therapy
 Correct other micronutrient deficiencies

What is the hemoglobin target for correction?

 Target for a hemoglobin level of 10 -12 g/dl. Avoid higher levels of hemoglobin as it has been
shown to increase cardiovascular risk
 Try to avoid the use of blood transfusions in these patients as much as possible, especially in
those who are planned for renal transplantation
How would you manage renal bone disease?

 The management of renal bone disease is based on the predominant cause

Recall the pathogenesis

 There are two predominant forms of renal bone disease


 High turnover disease – Due to decline of GFR and progressive renal dysfunction there is
retention of phosphate and decline in the activation of vitamin D. This causes low serum calcium
levels and high phosphate levels. This increases the activity of PTH and causes increased bone
resorption. High phosphate levels also increase secretion of FGF 23 from bone, which also
increases PTH, reduces level of active vitamin D
With time there is autonomous secretion of PTH resulting in tertiary hyperparathyroidism
 Osteomalacia can occur due to vitamin D deficiency
 The other form of bone disease is low turnover disease which is due to excessive suppression of
PTH due to excessive vitamin D and calcium supplementation

 Start with estimation of calcium and vitamin D levels, ideally with PTH estimation.
 Order X rays of the skull and hands to look for the classical features of hyperparathyroid
bone disease – pepper pot skull, sub-periosteal resorption
 Hyperphosphataemia is managed with phosphate binding resins such as calcium carbonate.
Calcitriol or 1-alpha calcidol should also be commenced
 Monitor the phosphate levels – keep them below 5.5 mg/dl

Mention that you need to monitor the calcium and phosphate levels very carefully

 Increase in calcium levels with Calcium carbonate and Vitamin D therapy can cause vascular
calcification and increase in cardiovascular events
 Total daily calcium intake should be limited to 1500mg/d
 It is also important not to drop PTH to very low levels, as this can cause adynamic bone
disease. This is because a certain amount of PTH is required to maintain the integrity of
healthy bone
 Therefore, the current trend is to use agents which do not cause significant increase in
calcium. Some agents such as Lanthanum carbonate are available in Sri Lanka
 Ideal management of renal bone disease requires regular monitoring of PTH levels
 PTH levels can be monitored in ideal settings. The target is between 2-9 times of the upper
limit of normal for the PTH assay
Jaundice

Background – highlight the relevant aspects of the medical background


of the patient

The age of the patient is important as it modifies your differential


diagnosis

Presenting complaint

The patient will present with yellowish discoloration of the eyes

Description of the presenting complaint

 Make sure that you do an extensive description of the presenting


complaint in chronological sequence
 Describe the onset, evolution and progression of the symptom
 Ask about the associated symptoms

Analysis of the presenting complaint

In the initial part of the history your main objective is to identify the
cause for jaundice. Recall that the causes of jaundice are classified into
pre hepatic, hepatic and post hepatic

Pre hepatic Hepatic Post hepatic


Causes Hemolysis Hepatocellular disease Obstruction
Disorders of uptake and Intrahepatic cholestasis
conjugation
Features Hemolysis Hepatocellular disease Prominent icterus
Prominent features of Icterus Dark urine
anaemia Dark urine Pale stools
Mild icterus Normal/dark stool color Pruritus
Dark urine and dark stools Systemic symptoms such
as nausea, vomiting and
features of hepatic failure

Inherited disorders of Intrahepatic cholestasis


uptake and conjugation Prominent icterus
Recurrent episodes Dark urine
History of neonatal jaundice Pale stools and pruritus

Cirrhosis
Long standing disease
with complications

Once you have established the profile of jaundice and classified it into one of the above categories,
ask further specific questions to identify the cause
A patient with hemolysis

Specific questions
Congenital hemolytic anaemia Ask for neonatal jaundice or childhood history of recurrent
hemolytic anaemia requiring blood transfusion
Gallstone disease or recurrent abdominal pain suggestive of
biliary colic (HS)
Exacerbation with certain medication and food items
(G6PD)
Family history of hematological disease and recurrent blood
transfusions (hemoglobinopathies)
History of consanguinity
Autoimmune hemolytic anaemia
Warm type Ask for associated features of autoimmune disease,
inflammatory type joint pain, alopecia, oral ulcers
Also ask for skin or mucous membrane bleeding suggestive
of autoimmune thrombocytopenia (Evan syndrome)

Cold type Ask specifically for acrocyanosis – bluish discoloration of


the extremities (should be differentiated from Raynaud’s
phenomenon – does not have the classical stages of
preceding pain, pallor, bluish discoloration and hyperemia)

Look for features of a secondary cause – recent or ongoing


respiratory symptoms (Mycoplasma)
Evidence of other cell line involvement, low grade
nocturnal fever with constitutional symptoms,
lymphadenopathy
History suggestive of retroviral illness
Blood transfusion related hemolysis Ask for recent blood transfusion
Mechanical thrombolysis Ask for past valvular surgical procedures
MAHA Preceding diarrheal illness with worsening AKI
Seizures
Confusion (TTP)
Paroxysmal nocturnal hemoglobinuria Ask for the early morning passage of dark urine and past
history of thrombotic events

Past medical history

Drug history – ask for use of medication that can precipitate hemolysis (Dapsone – G6PD,
methyldopa- hemolytic anaemia)
The patient with cholestasis

Specific questions
Viral hepatitis with cholestasis Ask for the classical preceding prodrome, characterized by nausea,
vomiting, malaise and fever
Look for risk factors for viral hepatitis in the history
Poor socio-economic status and epidemiological history
Ask for unsafe sexual contact and multiple sexual partners,
intravenous drug use
Autoimmune hepatitis/PBC Ask for associated features of connective tissue disease
overlap Oral ulcers, inflammatory joint pain, skin rashes and alopecia
Primary biliary cirrhosis Look back at the temporal profile of evolution. These patients will
have pruritus before the onset of jaundice
Also ask for dry eyes and dry mouth (associated Sjogren syndrome)
Primary sclerosing cholangitis Ask for associated chronic blood and mucus diarrhea and
seronegative arthritis with large joint arthritis and sacroileitis
HIV Ask for risk factors for retroviral illness
Lymphoma Ask for low grade nocturnal pyrexia, night sweats, constitutional
Associated porta hepatis nodes symptoms – loss of appetite and loss of weight
Paraneoplastic cholestasis
Drug induced A detailed drug history is essential
OCP, antiepileptics, recent therapy with antibiotics
Granulomatous disease Ask for features of sarcoidosis and past history of TB
Obstruction Evaluate and exclude obstructive causes for cholestasis
Ask for past history of abdominal pain suggestive of biliary colic
Biliary instrumentation in the past
Recent onset DM, severe abdominal pain with radiation through
the back, loss of appetite and loss of weight (pancreatic carcinoma)
Associated chronic diarrhea and steatorrhoea (chronic pancreatitis)

Hepatocellular

Viral hepatitis Ask for the classical preceding prodrome, characterized by nausea,
vomiting, malaise and fever
Look for risk factors for viral hepatitis in the history
Poor socio-economic status and epidemiological history
Ask for unsafe sexual contact and multiple sexual partners,
intravenous drug use
Autoimmune Ask for associated features of connective tissue disease
Oral ulcers, inflammatory joint pain, skin rashes and alopecia
Metabolic and toxin related Quantify recent use of alcohol, and exposure to other chemicals
disease Ask for family history of liver disease and consanguinity
Abnormal movements (Wilson’s disease)
Drug induced Take a detailed drug history
Vascular disease Ask for associated right hypochondrial pain and worsening
abdominal distension
Look for a possible history of susceptibility to thrombosis
Past medical history

Drug history – this is important due to the reasons detailed above

Gynaecological and obstetric history – this is important to detail in females and may indicate a
pregnancy related liver disease

Social history

 Discuss use of alcohol and other recreational substances


 Discuss sexual history

Examination

General examination

 Pallor
 Confirm the icterus
 Xanthelasma (PBC)
 KF rings
 Scratch marks
 Flaps
 Lymphadenopathy
 Peripheral stigmata of cirrhosis
 Evidence of chronic alcohol use
 Ankle edema
Cardiovascular and respiratory

 Perform a quick screening examination

Abdomen

 Look for hepatosplenomegaly


 Free fluid in the abdomen
Nervous system

 Features of hepatic encephalopathy


Assessment of abnormal liver function tests

Abnormal liver function tests 3. Isolated ALP – Exclude bone


origin – GGT and 5N

Conjugated
2. Global abnormalities
1. Isolated elevation of hyperbilirubinaemia
bilirubin
Direct > 15% of total

Unconjugated Dubin Johnson Hepatitic pattern Cholestatic


hyperbilirubinaemia Syndrome (hepatic pattern
pigmentation Higher
Direct < 15% of total proportional rise Higher
Rotor Syndrome in transaminases proportional rise
in ALP/GGT/5N
Vs. ALP
Vs. Transaminases
Or Mixed
Look for markers of Negative markers for Conjugated
hemolysis hemolysis Unconjugated/Mi
hyperbilirubinaem
xed
 NNC anaemia Urobilinogen (-) ia
 Elevated
reticulocytes
 Elevated LDH In adults consider Acute - > 500 ALP > 4x
 Low serum Gilbert Syndrome usually ALT > AST,
USS – duct dilation
haptoglbulin (mild elevation, ALP < 3x
 Urobilinogen + increased with stress,
AST > ALT –
starvation and use of
Hemolytic anemia alcohol) alcoholic hepatitis,
dengue, Ischemic
hepatitis

Chronic < 300,


ALT> AST, ALP < 3x

AST > ALT –


cirrhosis
Cirrhosis

A 36 year old male was admitted with massive hematemesis and had to be resuscitated with IV fluids
and blood. An emergency upper GI endoscopy was performed to arrest bleeding by banding on the
following day. He has had three similar episodes over the last one year. After the last bleed, he had
become drowsy and disoriented for 6 days from which he recovered fully. He had never consumed
alcohol and denies any family history of similar diseases. He had been investigated for darkening of
complexion and yellowishness of eyes one year back and was found to have diabetes as w ell.
Currently he is on insulin. On systemic review impotence for last 4 months and intermittent PR
bleeding for last 2 months were noted.

On examination the patient was lying with discomfort on the bed. He was pale and icteric with
generalized darkening of skin. Gynaecomastia, loss of body hair, palmar erythema, testicular atrophy
and bilateral ankle oedema was noted on general examination. There was an ejection systolic
murmur on the left sterna edge at 2nd intercostals space. The abdominal examination revealed a
splenomegaly (5 cm below costal margin) and gross ascites. The liver was not palpable. The nervous
system was clinically normal and the patient was conscious and rational.

Discussion

What is your diagnosis?

Chronic liver disease with portal hypertension

Why do you say so?

The patient in this case has presented with UGI bleeding. He also has stigmata of chronic liver disease
on examination. Splenomegaly and ascites are in keeping with portal hypertension. He also has
evidence of past episodes of hepatic encephalopathy

How would you evaluate this patient?

Investigations to confirm the diagnosis

 The diagnosis of cirrhosis is made on basis of the clinical evaluation and certain key
investigations
 Ultrasound scan of abdomen
 This will demonstrate a small nodular liver, enlarged left lobe and ascites. Newer imaging
modalities that measure liver stiffness (elastography) are also available
 Laboratory investigations
 FBC – anaemia, thrombocytopenia, pancytopenia (hypersplenism)
 Liver functions - Transaminases low/normal AST>ALT, Serum albumin – low, elevated serum
bilirubin, PT/INR high

Investigations to demonstrate the complications

 Upper GI endoscopy to evaluate for esophageal varices


 Diagnostic peritoneal tap in patients with ascites (SAAG)
 Renal function tests – hepatorenal syndrome
 Hepatic encephalopathy – serum ammonia, EEG – triphasic slow waves
 Echocardiogram – pulmonary hypertension – in porto-pulmonary hypertension
Investigations for the aetiology

The patient should undergo evaluation for a secondary cause based on the clinical evaluation. These
investigations include, serology for viral hepatitis B and C, autoimmune markers (ANA, Anti- Smooth
muscle) serum ferritin and transferrin saturation (hemochromatosis) and serum ceruloplasmin
(Wilson’s disease)

Discuss the principles of management

The aspects of management of a patient with cirrhosis includes definitive therapy for management
of the underlying cause and management and prevention of complications

General management

It is important to educate the patient on the avoidance of hepatotoxic drugs and other hepatotoxic
substances such as alcohol. Vaccination should be offered against hepatitis A and B.

Varices

Primary prevention of variceal bleeding

The patient should undergo upper GI endoscopy to identify varices. Pharmacological management
with beta blockers (propranolol) is the first line therapy. Repeat endoscopies should be planned
based on the initial extent of varices. Variceal band ligation should be offered in patients who have
contraindications to beta blockers. The practice of using ISMN as monotherapy or in combination
with beta blockers is not recommended

Discuss the management of acute variceal bleeding

This is a medical emergency. Hemodynamic stability of the patient is paramount at initial


presentation

 Insert two wide bore cannulae and start bolus infusions of crystalloids
 Medical management with intravenous terlipressin (2mg IV every 4 hours) or intravenous
octreotide (50µg bolus and 50µg/h infusion) should be commenced
 Arrange urgent endoscopy once the patient is stable. Several hospitals in Sri Lanka provide
24 hour endoscopy services. The endoscopy should be performed within 12 hours of
presentation
 Supportive management includes correction of coagulopathy with FFP and administration of
a broad spectrum antibiotic such as a 3rd generation cephalosporin/norfloxacin
 Continue the above medical management with vasoactive therapy and antibiotics for 3-5
days following onset of bleeding
 Blood transfusion should be performed with a hemoglobin target of around 8-9g/dl. Over
transfusion can increase the risk of bleeding
 Commence the patient on hepatic encephalopathy prophylaxis
 Other treatment modalities are used in patients with refractory bleeding. These are SB tube
insertion and TIPSS
Secondary prevention

 The patient should undergo variceal banding and ligation with beta blocker therapy as
secondary prevention. The patient who has repeated bleeds despite this should be
considered for TIPSS. This modality is available in Sri Lanka, and is performed by
interventional radiologists

Ascites

 The patient with ascites should undergo ascetic fluid analysis following diagnostic peritoneal
aspiration. Daily Na content should be less than 2g/d. No water restriction unless severe
hyponatremia (<130)
 Diuretic therapy - Spironolactone is the diuretic of choice (100 mg daily) together with
frusemide (40 mg daily). However monitoring of potassium is essential. The body weight is
an objective measure of the response to diuretic therapy. Target for a minimum weight los s
of 0.5kg/d
 Adjust doses of diuretics based on the serum potassium
 Discuss that you will be very careful with diuretics in patients presenting with upper GI
bleeding, hepatic encephalopathy or hyponatremia
 Diuretics should be stopped if the sodium values decrease to less than 125 and in the setting
of worsening hepatic encephalopathy
 Large volume paracentesis is required in patients with gross ascites. Post paracentesis
albumin is required when more than 5l are removed. 8g of albumin should be replaced with
every litre of fluid removed. TIPS is used in patients with refractory ascites

Spontaneous bacterial peritonitis

 Spontaneous bacterial peritonitis is a complication of ascites and is diagnosed with a


peritoneal fluid WBC of >/250 PMNL/mm. Antibiotic therapy with 3rd generation
cephalosporins should be given for 7 days with albumin infusions on D1 (1.5g/kg and 1g/kg
on D3 (especially in patients with renal dysfunction). All patients with an episode of SBP
require prophylaxis with norfloxacin.
 Also discuss there is a risk of SBP in patients on long term PPIs. Reconsideration for the need
for PPIs should be discussed and the drug discontinued if appropriate
Hepatic encephalopathy

 Correction of the precipitating cause is most important. Infection, dehydration and


hypokalemia are some of the common triggers
 Specific therapy
Lactulose should be administered with a target of 3-5 liquid stools per day. Use PEG if there
is no response with lactulose. Gut decontamination is done with oral rifaximin or oral
metronidazole. Other common precipitants are sedative (e.g. benzodiazepines) and acute GI
bleeding
Branched chain aminoacids and oral/ intravenous LOLA may also have a place in the
management
Make sure that the nutritional state of the patient is managed carefully
Liver transplantation

 Liver transplant is offered in some tertiary care centers in Sri Lanka for the management of
patients with cirrhosis. Patients are selected using scoring systems such as Child-Pugh score
and MELD score

Hepatoma

 All patients with cirrhosis need to be monitored for the development of hepatocellular
carcinoma. The main investigations used are alpha feto protein and USS
Diabetes mellitus with complications

This is probably the most common long case to expect at the exam

History

Presenting complaint

 Describe the presenting complaint in detail and in chronological sequence

Past medical history

 List the other associated diseases


Diagnosis

 Diagnosis – discuss how the diagnosis was established


 Obtain enough details to determine the type of DM – determining LADA and MODY from
type I and II can be challenging
 Describe the initial therapy

Describe the complications of the disease

 Episodes of hyperglycaemia requiring hospital admission


 Recurrent infections – especially skin and subcutaneous tissue infection
 Macrovascular complications – ischaemic heart disease, cerebrovascular disease, peripheral
vascular disease
 Microvascular disease – retinal disease, nephropathy, neuropathy – including autonomic
neuropathy – nocturnal diarrhea, gustatory sweating
 Describe sexual dysfunction and effects
 Foot disease

Describe the treatment history of the disease

 Outline the medication used from the time of diagnosis to the current admission
 Describe the adverse effects of medication
 Make a special mention of hypoglycaemic episodes – both major and minor hypoglycaemic
episodes
 Ask specifically on insulin therapy. If the patient is on insulin focus on the specific points
given below
Reason for commencement of insulin
Type of insulin prescribed
Technique of administration and the person who administers it
Patient education of use of insulin and relationship between meals and exercise
Hypoglycaemia associated with insulin therapy
Compliance to therapy
Storage of insulin
Disposal of needles
Complications of lipoatrophy and lipodystrophy
Monitoring and control

 Ask of the availability of a glucometer for self-glucose monitoring


 Mention the values of recent HbA1C, FBS and RBS

Follow up

 Describe the institute of follow up and the compliance of the patient


 Last screening investigations for organ damage

Social history

 Discuss the patient education on the disease and the disease impact on the patient’s life
 Highlight aspects on foot care

Examination

General examination

 Weight, height, BMI, waist and hip circumference


 Features of metabolic syndrome
 Examine the insulin injection sites for complications
 Examine the skin for diabetes related complications – scleroderma diabeticorum, granuloma
annulare, necrobiosis lipoidica diabetecorum

Cardiovascular

 Blood pressure
 Check for postural hypotension

Abdomen

 Hepatomegaly

Fundus

 Diabetic retinopathy

Cases summary

65-year-old father of two children, presenting with painful localized swelling of bilateral upper legs
for 3 days duration associated with high grade fever with chills and malaise. He has a background
history of T2DM for 4 years complicated with macro and microvascular complications. DM -
incidental finding in May 2016 following investigation for a non healing wound. Osmotic symptoms
were present at that time with weight loss. He was commenced on metformin, gliclazide,
atorvastatin, losartan, aspirin with monthly follow up. He has a history of IHD and is awaiting CABG.
In addition, the patient has features of painful symmetric diabetic neuropathy. He is p oorly compliant
with medication and has a poor knowledge of the disease. Examination is suggestive of bilateral
lower limb cellulitis and peripheral neuropathy. His BP is 190/80 and fundus shows non proliferative
retinopathy
How would you optimize the management of this patient?

 The first step is to review the diagnosis of the patient. The criteria for the diagnosis of DM
are presented below

Discuss that your first step would be to review the diagnosis of the patient

Definition and classification

DM is defined based on the


fasting blood glucose, OGTT (2
hour plasma glucose after a
glucose challenge) and the HbA1c
values

DM is classified into four groups

 Type 1
 Type 2
 Other specific types
MODY and LADA
LADA is a late onset from of immune mediated DM, in which patients require insulin therapy
within the first 6 months of diagnosis. The age of onset is variable and may be beyond 30
years. These patients have autoantibodies in their serum
MODY – these patients have young onset DM, with a strong family history ( AD) and are
managed with oral hypoglycaemics. Antibodies are negative. They do not have overt
evidence of insulin resistance
 Gestational diabetes mellitus

Type 1 Type 2
Epidemiology Age of onset < 30 years All racial groups – incidence increasing rapidly
worldwide – especially in Asian and African
populations
Pathogenesis Autoimmune beta cell destruction Insulin resistance and abnormal insulin
secretion
Genetic predisposition – HLA DR3 and DR4 Strong genetic association
positive in majority Insulin resistance occurs as a consequence
Infiltration of pancreatic beta cells by obesity and genetic susceptibi lity
lymphocytes Insulin resistance causes increased hepatic
glucose output and reduced peripheral uptake
of glucose by skeletal muscle and adipose
tissue

Pancreatic beta cell destruction occurs with Reduced insulin secretion occurs in later
the action of cytokines and T cells stages due to beta cell failure
Antibodies to pancreatic islet cells are also
present

Clinical Younger, non obese (may los e weight) Usually asymptomatic in early stages. May
Present with DKA present with osmotic features
Features of other autoimmune disease Obese, have features of metabolic syndrome
Management Absolute insulin requirement Require insulin in later stages
Optimizing the management of Type II DM

This is a very important aspect of the management plan. The patient should be educated regarding
the following aspects

 Education regarding the disease and complications


 Education regarding the diet and lifestyle modifications that are essential for good control of
blood glucose and prevents complications
 Advise patients on the importance of compliance to therapy, often life-long even in the
absence of symptoms.
 Educate patients on the adverse effects of therapy – especially on hypoglycemia
 Training on using insulin is required in patients commenced on insulin therapy
 Educate the patient on detecting and preventing complications – particularly on DM foot
care

Monitoring of glucose

The patient should be given the option of self –blood glucose monitoring. This is currently expensive,
but some patients may prefer this option. Discuss that patients on intensive insulin regimens will
benefit from this method of evaluation

Management of associated comorbidities

Special attention should be made towards management of other cardiovascular comorbidities in


patients with DM including hypertension, hypercholesterolemia and ischaemic heart disease

Physical activity and lifestyle modifications

The patient should be encouraged to engage in moderate aerobic exercise for about 3 and ½ hours
per week spread out over around 3 to 5 days. Stopping smoking is essential.

Diet in Type II DM

Discuss the following aspects if asked specifically on the diet in DM

 The diet of a patient with type II DM should be balanced. The patient should be advised that
skipping meals should be avoided
 A general structured meal plan includes 3 main meals and 2 snacks per day. A large meal will
cause an increased calorie load and hyperglycemia. Therefore, what is recommended is to
have small frequent meals (i.e. 3 meals with 2 snacks). As obesity is a frequent concomitant
problem reduction of portion sizes per meal is extremely important.
 The principle is to eliminate refined carbohydrates (as they cause a rapid increase in blood
glucose), minimize carbohydrates per meal, replace meals with fibre containing leafy
vegetables and reduce meat. Fish and white of egg are good sources of proteins.

Carbohydrates

 Carbohydrates should constitute 45-65% of total intake.


 Low glycaemic index, fiber containing food should be encouraged. Parboiled or minimally
milled rice, whole grain cereals and flour are recommended.
 Non starchy vegetables can be used without restriction.
 Consumption of refined sugars such as cakes, sweets and fi zzy drinks should be avoided or
minimized. The quantity of carbohydrates should also be regulated. Certain accepted
methods such as the food plate method and carbohydrate counting method can be utilized

Fat

 Fat should constitute 20 -35% of total caloric intake


 The use of saturated fat should be limited. Unsaturated fat containing foods such as fish and
nuts may be used. Unsaturated cooking oil are preferred
 Re-using vegetable oils for frying is to be avoided because with heat, the unsaturated fats
become saturated. The patient should be advised against using food rich in trans-fat (seen in
many margarines).

Protein

 Protein should constitute 10-35% intake. Plant protein sources and fish can be utilized. Red
meat contains significant fat and must be used in moderation

Fruits

 Half ripe fresh fruits may be consumed. Some fruits such as bananas, mango, papaya and
pineapple have a high glycaemic index and should be consumed in moderation. Fruits that
are sour tend to be suitable (e.g. woodapple, naarang, etc)

Pharmacological management and optimizing therapy

Initial therapy

The patient should be commenced on metformin as initial therapy with concomitant lifestyle and
dietary modifications. Some patients with very high glycaemia at presentation may require dual
therapy with SU, or initiation of insulin

Subsequent therapy

If glycaemic targets are not met despite treating with metformin at highest tolerated doses (to a
maximum of 1 g tds) for 3 months, another agent may be added. Options for dual therapy are,
sulphonylureas, DPP 4/GLP agonists, TZD or basal insulin (long acting). In Sri Lanka, the most cost
effective drug to use is a sulphonylurea
Drug class Mechanism of Advantages Adverse effects Contraindications
action
Biguanides Decreases Extensive experience Gastrointestinal Major organ failure
Metformin hepatic glucose Does not cause side effects – A dose reduction is required
output hypoglycaemia nausea and when the GFR < 45 and the
Reduces insulin Induces weight loss vomiting drug should be discontinued
resistance Has beneficial effects(these can be when GFR < 30
on the lipid profile limited with the
use of extended Metformin should also be
Decreases release avoided in patients with
cardiovascular preparations) severe heart failure, liver
mortality Lactic acidosis failure and acidosis
Reduces microvascular Vitamin B12
complications deficiency
Sulphonylurea Stimulation of Extensive experience Weight gain Renal/liver disease
Tolbutamide insulin release Decreased
Glibenclamide from beta cells microvascular Hypoglycaemia
Gliclazide through complications – higher risk
Glimipiride closure of K+ with starvation,
ATP channels alcohol and
renal failure
Metabolized in
the liver and
excreted via the
kidney – longer
acting agents
(glibenclamide)
to be avoided in
elderly and in
renal/liver
impairment
Interactions –
warfarin,
ketoconazole

Meglitinides Stimulation of Short acting, Weight gain Renal/liver disease


Repaglinide insulin release decreased post and
Nateglinide – closure of K+ prandial glucose hypoglycaemia
ATP channels fluctuations
Thiozolidinediones PPAR –gamma No hypoglycaemia Weight gain Cardiac failure
nuclear Increases HDL –C Edema and Renal failure and liver
receptor Reduces TG worsening of disease
activation cardiac failure Osteoporosis with long term
Reduce insulin Increased CVD use
resistance, with
redistribution rosiglitazone,
of fat Increased LDL
Hepatic damage
Osteoporosis
and fractures
GLP 1 receptor agonists Incretin No hypoglycaemia, GI side effects Renal disease
Exenatide Amplify weight reduction Medullary Medullary carcinoma of the
glucose Improves post carcinoma in thyroid
stimulated prandial glycaemic animal studies
insulin control Acute
secretion Cardiovascular pancreatitis
Decrease protection
glucagon
Reduces gastric
emptying
DPP 4 antagonists Increase GLP – No Limited Renal disease (reduced
Sitagliptin 1 hypoglycaemia/weight experience dose)
concentration gain Urticaria and
Increase Improves post angioedema
glucose prandial glycaemic Pancreatitis
dependant control
insulin
Decreases
glucagon
Alpha glucosidase Inhibits No hypoglycaemia Gastrointestinal Inflammatory bowel disease
inhibitors intestinal alpha Minimal side effects adverse effects Renal disease
Acarbose glucosidase – Improved post
slows prandial glucose
carbohydrate control
digestion and
absorption
SGLT – 2 inhibitors Selective Cardiovascular Urinary tract
Canaglifozin inhibition of protection infection
Empagliflozin the sodium Vaginal
glucose infection
contransporter Euglycaemic
in the kidney ketoacidosis

Insulin therapy

 If the patient fails to reach glycaemic targets with two agents within 3-6 months, a third oral
drug may be considered. The choices include DPP IV inhibitors, TZD and SGLT 2 inhibitors
(currently not available in Sri Lanka) However initiation of insulin therapy is probably the
best option in these patients. GLP – 1 agonists are emerging as alternates to insulin
 Discuss that you will start with basal insulin therapy at bedtime (isophane insulin or long
acting insulin analogues – detemir, glargine). Start with a dose of 0.1-0.2 U/kg/d, but higher
doses can be used in patients with extremely poor glycaemic control. Increase the dose with
close CBS monitoring by increments of 1-2U
The longer acting insulin analogues have greater predictability of action, less we ight gain and
risk of hypoglycaemia
What will you do if the patient has persistent poor control of HbA1C?

 Soluble insulin at meals can be added to achieve prandial control, if glycaemic targets are
not met within 3-6 months. However this can increase the number of injections per day
 An alternative is to combine GLP-1 agonists with basal insulin, but this option may not be
feasible for most Sri Lankan patients
 Pre-mixed insulin can be administered 2-3 times a day, with 60% of the dose given in the
morning hours
 Premixed insulin contains a ratio between a short acting insulin and intermediate/long
acting insulin (30/70, 25/75, 50/50). In premixed insulin available in the government sector
the composition is soluble insulin and isophane insulin. Newer preparati ons contain rapidly
acting insulin analogues such as aspart and lispro as the short acting component
 Adjust the doses by increments of 2-4 based on the CBS values
 A basal bolus regimen of soluble insulin with all meals and basal insulin is commenced in
patients who are difficult to control with the above regimen

What is the advice you would give a patient on insulin therapy?

 Reinforce the previous education


 Tell the patient the reason for starting insulin
 Advise on where to obtain insulin and the insulin injection devices
 The most commonly used device will be a plain syringe – this has a 29G needle and should
be calibrated up to 100U of insulin
 Storage of insulin – in the refrigerator (middle compartment)
 Before injection have a wash. Check the injection bottle (regular insulin is colorless and all
other preparations are turbid)
 Gently roll the bottle on your palms
 Do not use surgical spirit to clean the area
 Demonstrate the technique of injection
 Tell the patient to inject on slightly different places in the same site and rotation of the sites
 Syringes can be reused if the same person is using it. Dispose sharps into a sharps bin
 Have your meals to avoid hypoglycaemia
 Educate the patient on the complications of insulin therapy

How would you set glycaemic targets for this patient?

The general glycaemic targets for Type II DM are as follows

 A1C <7.0% (5.3 mmol/L)


 Preprandial capillary PG 80-130 mg/Dl (4.4-7.2 mmol/L)
 Peak postprandial capillary PG <180 mg/dL (10.0 mmol/L)

However, less stringent values are recommended in patients with

 History of severe hypoglycaemia


 Limited life expectancy
 Advanced microvascular or macrovascular complications
 Comorbidities

Discuss how you would manage the cardiovascular complications and risk factors in this patient?

 Manage cardiovascular risk


 The major cardiovascular risk factors are,
LDL-C ≥100 mg/dL (2.6 mmol/L)
High blood pressure
Smoking
Overweight or obesity
Family history of premature ASCVD
 Hypertension management – manage to a target of < 140/90, a lower target of 130/80 is
recommended in young patients, patients with albuminuria and those with additional
cardiovascular risk factors
Start lifestyle and dietary modifications. ACEI or ATRB are the treatment of choice
 Lipids

The patient should be started on high dose


statin therapy if,

DM and past/current atherosclerotic


event, regardless of the age
Age > 40 with cardiovascular risk
factors

Moderate intensity statins


Should be considered in patients > 40
years even if they do not have
additional cardiovascular risk factors

Note:
The addition of ezetimibe to moderate-intensity statin therapy has been shown to provide
additional cardiovascular benefit compared with moderate-intensity statin therapy alone,
and it may be considered for patients with a recent acute coronary syndrome and a low-
density lipoprotein cholesterol level of 1.3 mmol/L (50 mg/dL) or greater or for those who
cannot tolerate high-intensity statin therapy
Ezetimibe inhibits absorption if cholesterol in the small intestine

In general, combination of statins and fibrates are not useful except for management of
severe hypertriglyceridemia
 Antiplatelet therapy may be considered in patients who have high cardiovascular risk for
primary prevention. This generally includes men > 50 years and women > 60 years with
additional cardiovascular risk factors
 Initiate regular screening programs
What are the problems you would anticipate in patients on statin therapy?

 Myositis and rhabdomyolysis are common issues. The risk is increased in patients who are
hypothyroid
 Elevated transaminases can be another issue. Elevation of transaminases more than 3-5 ULN
is an indication to stop statin therapy

The patient has recurrent hypoglycaemic events. What will you do?

 Evaluate the patient for evidence of


hypoglycaemia unawareness and autonomic
failure
 Autonomic failure in DM reduces the
autonomic and adrenomedullary responses to
hypoglycaemia resulting in hypoglycaemia
unawareness and defective glucose counter-
regulation. Episodes of hypoglycaemia
worsens the autonomic failure
 Adjust insulin levels
 Manage hypoglycaemia immediately with
boluses of 50% dextrose if the patient is
acutely ill
 In patients with hypoglycaemia unawareness,
a higher glycaemic target should be discussed
 If the patient has nocturnal hypoglycaemia, advise on the use of a bed-time snack and
reduce the dose of long acting insulin. Long acting insulin analogues have a lower risk of
hypoglycaemia

Discuss the management of microvascular complications in your patient

Diabetic retinopathy

 Refer the patient immediately for ophthalmologist review if there is evidence of proliferative
DR or diabetic maculopathy
 Modify all vascular risk factors. Intensive glycaemic control reduces progression
 Laser photocoagulation therapy indicated to reduce the risk of vision loss in high-risk PDR
and severe NPDR
 Intravitreal injections of antivascular endothelial growth factor are indicated for center-
involved diabetic macular edema
 Discuss that you will arrange for regular eye reviews

Diabetic nephropathy

 Screen the patient with microalbuminuria or albumin/creatinine ratio


 Presence of > 300mg/d of persistent albumin excretion in a patient is suggestive of DM
nephropathy. Microalbuminuria is defined as albumin excretion of 30-300mg/d
 ACEI/AT II RB should be commenced in these patients
 Optimizing the glycaemic control and cardiovascular risk factors are of extreme importance
 HbA1C monitoring is inaccurate in patients with progressive renal disease and CBS
monitoring and FBS, PPBS measurements become more important
 SU should be used with caution in patients with renal dysfunction (glibenclamide should be
avoided). Gliclazide is safe to use. DPP 4 inhibitors should be reduced in dose with declining
GFR. Pioglitazone should not be used
 Metformin should be discontinued when the GFR declines below 30

Neuropathy

 There are several types of diabetic


neuropathy
 The patients should be evaluated for
neuropathy using a 10g monofilament and
also for vibration sense using a 128Hz
tuning fork
 Pharmacological agents used for the
management of painful neuropathy include
TCA, SNRI (duloxetine), anticonvulsants
(Gabapentin)
 Proper foot care should be emphasized

Autonomic neuropathy

 Several issues should be addressed in the management


 Postural hypotension
 Exclude other causes in your clinical and laboratory evaluation – drugs, dehydration,
Addison’s disease
 Stop all medication precipitating postural hypotension
Advise patients on getting up slowly from the seated position
Increase salt and fluid intake
Raising the head end of the bed
In difficult to manage cases pharmacological therapy should be used – this includes
fludrocortisone, and in severe cases ephedrine
 Diabetic gastroparesis
This commonly presents with diarrhea
Exclude other causes of diarrhea in patients with DM – bacterial overgrowth, exocrine
pancreatic insufficiency, drugs (metformin), coeliac disease in patients with Type I DM
Adjust medication – drugs that delay gastric emptying (incretins), adjust doses of metformin
Use prokinetics – domperidone, metoclopramide
Erythromycin
How would you address the problem of sexual dysfunction in this patient?

 Erectile dysfunction is the most common disorder of sexual function in a patient with DM.
This is due to contributions from vascular disease, as well as autonomic neuropathy
 The patient should be referred to a counselling service to address the psychosocial
implications of the disease
 Pharmacological therapy
 PDE inhibitors – sildenafil, vardenafil (4-6 hours) and tadalafil (36 hours). Make sure the
patient is not on concomitant nitrate therapy, nicorandil or nebivolol
 Other therapies
Rheumatoid arthritis

History

Introduction

Background

 The patient will probably be a diagnosed patient with Rheumatoid arthritis and the focus will
be on management issues

Presenting complaint

 This will usually be with an acute flare of the disease


 Describe the onset and progression of the disease with the associated features
 Describe the current state of functional restriction

Past history

 Go back to the onset of the disease


 Describe the initial presentation of the patient and how the diagnosis was made. Describe
the results of relevant investigations at that time – markers of inflammation, rheumatoid
factor
 Ask for other evidence suggestive of associated connective tissue disorders (malar rash,
photosensitivity, oral ulcers, muscle aches and pain with proximal muscle weakness, skin
thickening, Raynaud’s phenomenon) – this is important as RA can coexist with these CTDs
 Describe the initial management of the patient and follow up
 Highlight the complications of the disease over time – use a time line

Important features
Skin  Ask for ulcers (Pyoderma gangrenosum)
 Other rashes
Cardiac  Ask for history of pleuritic chest pain or chest pain suggestive of
pericarditis
 Ask for past history of myocardial infarction or ischaemic heart
disease
 Ask for features of left ventricular dysfunction
Pulmonary  Ask for progressive shortness of breath (interstitial lung
disease)
 Past history of pleural effusions
Eye  Ask for dry eyes and dry mucus membranes (Sjogren syndrome)
 Ask for red eyes, grittiness in the eyes, visual disturbances
(scleritis, episcleritis)
 Ask for past history of ulcers
Neurology  Neck pain with radiation to the occipital region with difficulty in
walking and upper limb weakness (AA subluxation)
 Progressive lower limb numbness suggestive of peripheral
neuropathy
Hematology  Ask for features suggestive of anaemia (dyspnea, lethargy and
malaise)
 Ask for recurrent sino-pulmonary infections, skin sepsis
Renal  Ask for progressive lower limb swelling and persistent frothy
urine
Bones  Ask for recurrent fractures – specially with trivial trauma
Describe the medication history

 Using the same timeline describe the medication history and important changes done over
time
 Ask for the adverse effects of medication
 Ask if the patient has been considered for administration of biologicals

Drug Important points


NSAIDS  Ask for gastrointestinal adverse effects
Steroids  Can also cause GI adverse effects
 Ask about weight gain
 Ask about the last FBS
 Ask about eye complications (cataract)
MTX  Does the patient have evidence of reduced neutrophil
counts and other cytopenias
Once weekly administration
Look for renal function and hepatic function monitoring
and recent alteration
SSZ  Skin rashes, alteration in blood counts, issues with fertility
in males (oligospermia)
HCQ  Ask for visual disturbances and when the last eye review
was performed
Leflunamide  Ask for worsening diarrhea, ask if the patient is aware of
the teratogenic potential and if necessary precautions have
been performed
Discuss the functional capacity of the patient

 Use the activities of daily living (ADL) and instrumental activities of daily living (IADL) as a
guide

Past medical and surgical history

Drug history

 This has been described above


Social history

 Describe the personal details of the patient


 Discuss the impact of the disease on the patient on the person and family
 Discuss the support available to the patient
 Describe the household environment with relations to the limitations of activity
Joint examination

In a patient with RA, it is important to perform a complete joint examination

 On inspection look for the typical joint deformities of RA


 These include – swan neck deformities, boutonniere deformities, ulnar deviation, Z shaped
thumb
 Look for joint swelling and examine for joint line tenderness and synovitis
 Perform a range of movement and functional assessment

Case summary

A 43 year old widow and mother of 3 children presented with B/L symmetrical pain and swelling of the
metacarpophalangeal and proximal interphalangeal joints with associated morning stiffness lasting
for more than 1 hour. She also complains of B/L shoulder and knee joint pain. She denies alopecia, skin
rashes or oral ulcers. She also complains of symptoms suggestive of anaemia and peripheral
neuropathy for 2 months duration. She has poor social support and her functional status is poor.

Her examination is significant for pallor, B/L neck deformities and swelling of the MCP and PIP joints.
Examination of the nervous system revealed sensory loss consistent with a polyneuropathy.

Discussion

What is your diagnosis?

Rheumatoid arthritis

Why do you say so?

When formulating a differential diagnosis for a patient with joint pain the following aspectsshouldbe
considered (see also approach to polyarthritis)

 Age of the patient


 Duration of symptoms
 Number of joints involved and pattern of involvement
 Inflammatory or non inflammatory process
 Involvement of other systems

In the case given above this middle-aged woman have features of a symmetrical polyarthritisaffecting
both large and small joints. The DIP, spine and axial skeleton are not involved. The symptoms have
lasted for 5 weeks. Early morning joint pain with stiffness lasting for more than 1 hour and featuresof
inflammation (redness, heat and swelling) are in favor of an inflammatory process. There is no
evidence of extra articular involvement.

D/D:

 Rheumatoid arthritis

 SLE – Is less likely due to lack of cutaneous manifestations or organ involvement

 Spondyloarthopathies – Is less likely due to lack of involvement of the spine


 Generalized OA – Is less likely due to the inflammatory process and pattern of joint
involvement (sparing of the DIP joint)

Discussion on RA
How would you establish a diagnosis of
rheumatoid arthritis?

 The diagnosis of RA is generally


clinical, in a patient with
symmetrical inflammatory
polyarthritis
 When patients have atypical
presentations, classification
criteria can be used to aid the
diagnosis
 These criteria involve clinical,
serological and laboratory
criteria
 Make sure to request
serological investigations with
titers

What are the investigations you would like to perform in this patient?

 To establish the diagnosis, review the inflammatory markers, rheumatoid factor/CCP and
hand X rays
 The classical changes that occur in the X - ray are periarticular osteopenia, symmetric joint
space loss – seen more prominently in the wrist as crowding of carpal bones, subchondral
erosions, especially at the MCP and PIP in the free surfaces. Overt rheumatoid deformities
are rare in early disease
 Other investigations may be required to screen for organ complications and extra-articular
disease. Discuss these based on your clinical assessment
 Perform baseline investigations before commencing drug therapy – FBS, liver function test,
renal function tests

How will you treat the patient?

General management

 Rheumatoid arthritis is a chronic disease and requires significant patient motivation.


Therefore, the first step in the management should be to counsel the patient regarding
disease and educate them on the importance of proper compliance
 Refer the patient for physiotherapy and occupational therapy, some patients require splints
to overcome deformities and to improve their functional capacity
 Make sure you discuss the management of other comorbidities, especially cardiovascular
risk factors, as RA confers increased cardiovascular risk
Pharmacological management

 It is essential in the management of RA, to commence DMARD therapy as early as possible


and to treat to a target of remission. This has been shown to prevent adverse outcomes and
limit the structural progression of the disease. This patient should have been started on
early DMARD therapy at the point of diagnosis. The DMARD of choice as initial therapy is
MTX due to wide ranging benefits
 Mention that the goal of therapy is to reduce the disease activity to minimal levels as soon
as possible
 Methotrexate is the first line drug for use in RA. Leflunomide or SSZ may be used if the
patient has contraindications for MTX
 Steroids are used as bridging therapy in patients with RA. Steroid can be used in oral
courses, intramuscular/IV. A short course is recommended.

How would you monitor a patient on MTX?

 The patient should be instructed on the proper dose of MTX and the method of
administration
 Monitor the patients FBC and liver function tests (FBC, serum creatinine/ SE and LFT every 2
weeks for about 6 weeks into the therapy. The frequency can be reduced to monthly after
this)
 Monitor for MTX induced lung disease. This usually occurs within 1 year of commencing
therapy. It usually causes an acute – subacute pneumonitis. The risk is higher in patients
with pre-existing lung disease
 The acute or subacute pneumonitis associated with MTX is usually associated with fever,
cough and shortness of breath and diffuse pulmonary opacities. The HRCT may demonstrate
diffuse/patchy ground glass opacification and nodules. BAL shows predominant lymphocytes
 Warn the patient on common adverse effects such as nausea, vomiting and hair loss
 If the patient is in reproductive age, advise on contraception as MTX can cause fetal
malformations
 Patients should continue contraception until > 3 months off MTX
 MTX also causes hepatic toxicity leading to hepatic fibrosis and cirrhosis. This depend s on
the cumulative dose of MTX as well as the weekly dose
 It is safe to omit MTX and ask for expert opinion when WBC < 3.5 and liver enzymes rise
more than 2x upper limit of normal

Pharmacological management – continued

 Perform regular follow up of the patient. MTX generally takes around 3-4 weeks to start
acting
 Bridging steroid therapy (prednisiolone - 7.5-10mg) can be used and later tailed off
 It is important to achieve symptom
relief within 6 weeks, titrate the dose
and achieve targets within 6 months. If
treatment targets are not achieved
within this period, second line therapy
will be required
 Guidelines recommend that biologicals
should be started as second line
therapy if the patient has poor
prognostic factors such as extra-
articular disease, but this may not be
practical in Sri Lankan patients. In our
setting combinations of MTX and other
conventional DMARDs such as SSZ and
leflunomide are used
 In Sri Lanka biologicals are used in
tertiary care rheumatology clinics
when the patient does not respond to
combination DMARDs

What are the other important issues in the management of this patient?

 Anaemia
This is one of the most common problems in patients with RA. This is most often due to
anaemia of chronic disease, but other factors such as, iron deficiency anaemia due to
inadequate diet, chronic upper GI bleeding as side effects of medication – NSAIDs and
steroids, folate and B12 deficiency also play a role
Rare possibilities include bone marrow suppression and LGL
Make sure you discuss a proper evaluation of anaemia using the full blood count, blood
picture and iron profile
 Pulmonary disease
This is also a very common management problem. Start investigations with a CXR and lung
function test, and move on to HRCT
The most common HRCT findings are of UIP with predominant basal sub pleural involvement
with honeycombing. Lung function test will show a restrictive pattern
How would you address the problem of steroid toxicity in this patient?

 This can be a major problem in all rheumatological disease. It is important to review the
patient’s fasting blood sugar and perform an eye review for cataract
 Steroid induced osteoporosis requires further evaluation with DEXA scan
Patient who are post- menopausal and are on glucocorticoid therapy > 7.5mg for 3 months
or more may be commenced on Bisphosphonate therapy based in their risk for an
osteoporotic fracture
Evidence is less clear for patients who are premenopausal unless they have a past fragility
fracture and are on steroids for > 3 months. In these patients a DEXA scan needs to be
performed to assess the need for Bisphosphonate therapy
 Supplement patients with calcium and vitamin D
 These patients are also at a high risk of steroid induced avascular necrosis
How would you follow up this patient?

 The patient should ideally be referred to a rheumatologist for continuing care


 At each clinic visit, it is important to assess the patient’s joint score. There are several ways
of doing this including the CDAI, SDAI and DAS 28 scores. The CDAI is widely used as it is a
clinical score
 Assess the patients quality of life and restrictions to daily activities
 Assess the adverse effects of medication and make necessary adjustments
 Set targets for the next visit

The patient complains of severe neck pain. What do you think? How will you evaluate?

 Consider the possibility of atlanto-axial subluxation


Backache

Introduction to the patient and relevant background

Presenting complaint

Description of the presenting complaint

 Consider the age of onset – a younger age of onset may indicate a spondyloarthopathy
Recent onset pain in patients more than 50 years of age may indicate a sinister cause
 Describe the onset and progression of pain – especially highlight new onset progressive
backache
 Describe the immediate preceding state of the patient – ask for episodes of trivial trauma
 Ask for previous similar episodes
 Describe the pain
Location – thoracic pain of recent onset may indicate a sinister cause
Quantification and assessment of severity
Radiation of pain – along the lower limbs – this may indicate nerve root compression
Ask for exacerbating factors – walking and activity, coughing and straining
Ask for relieving factors
 Ask for associated features of the pain – focusing on the following
Alarm symptoms

 Predominant thoracic pain


 Associated fever and night sweats
 Loss of appetite and loss of weight
 Lack of response to simple analgesics
 Nocturnal pain
 Associated neurological symptoms indicating cord compression – urinary retention, bladder
and bowel incontinence, numbness in the perineal region, difficulty in walking, associated
lower limb weakness

Features suggestive of a spondyloarthropathy – these symptoms are mostly important in young


patients with backache

 Backache worse in the morning with associated morning stiffness


 Reduced range of movement of the spine
 Associated asymmetrical large joint arthritis with associated stiffness

Ask specific questions to evaluate the aetiology

Sinister backache
Infections
Pyogenic vertebral osteomyelitis Recent hospital stay, intravenous antibiotic therapy

TB Past history, or history of close contact to TB, constitutional


symptoms, sacroiliac joint pain, associated pulmonary
involvement
Brucellosis Consumption of unpasteurized milk, working in, or originating
from a farming community. The presentation may mimic TB
Malignancies
Hematological
Myeloma Associated features of anaemia, recent clinical features of
renal dysfunction, neurological involvement, features of
hypercalcaemia such as constipation, polyuria and polydipsia
and mental confusion
Amyloidosis – skin changes, associated peripheral neuropathy,
features of autonomic neuropathy such as postural instability
Ask for feature of hyperviscosity – blurring of vision, recent
onset headaches

Secondary vertebral deposits Ask for features of a possible primary site – breast in females,
renal cell carcinoma – hematuria, thyroid lumps, prostate –
lower urinary tract symptoms, bronchial – cough with
hemoptysis
Inflammatory Spondyloarthropathy
Ask for associated red eye, visual impairment, photophobia
and eye pain
Enthesitis
Skin rashes and nail changes (psoriasis)
Associated bowel symptoms, chronic diarrhea, blood and
mucus diarrhea (enteropathic arthritis)
History of urethritis (Reactive)
Degenerative
Osteoporosis with fractures Ask for features predisposing to osteoporosis –
rheumatological disease, steroid use

Describe the current state of investigations and management

Past medical and surgical history

 Describe other comorbidities in detail


Drug history

 Focus on long term use of medication – especially on steroid use and NSAIDs. Focus on the
complications of these medications

Social history

 Use the social history to describe the functional limitation of the backache in detail. Describe
the impact on the patients ADLs and IADLs
 Discuss the consequences of the backache in relation to the occupation
 Describe the environment of the house and the workplace – highlight the limitations the
patient has due to his/her symptoms
 Discuss the psychological impact of these symptoms
Examination

General examination

 Pallor – may indicate a possible hematological cause


 Look for cutaneous stigmata amyoidosis – periorbital ecchymosis, pinch purpura, waxy
plaques macroglossia
 Lymphadenopathy
 Skin for intravenous injection marks
 Thyroid examination – if secondary deposits are considered
 Ankle edema

Examination of the back

 Look for local points of tenderness, masses


 Examine the sacroiliac joints for tenderness
 Perform the modified Schober test and the wall tragus test if the patient is suspected to
have a spondyloarthropathy

Cardiovascular

 Perform a quick cardiovascular examination – look for evidence of restrictive


cardiomyopathy (amyloidosis)
 AR may be noted in patients with spondyloarthopathy
 Measure postural drop – autonomic neuropathy in amyloidosis

Respiratory

 Look for pleural effusions and other focal lung signs

Abdomen

 Hepatosplenomegaly is an important finding and may indicate amyloidosis. Organomegaly


may also be a feature of chronic infections such as brucellosis
 Perform a digital rectal examination to assess the prostate and also a quick examination of
the external genitalia for masses

Nervous system

 Examine the nervous system in detail


 Look for clinical evidence of cord compression – cauda equina syndrome, thoracic cord
compression
 Straight leg raising test when there is evidence of nerve root compression
 Fundus – evidence of hyperviscosity
Discussion point

What are the key features of Spondyloarthropathy?

 Predominant axial involvement – spine and sacroiliac joints


 Inflammatory back pain
Age at onset, <45
Duration, >3 months
Insidious onset
Morning stiffness >30 min
Improvement with exercise
No improvement with rest
Awaking from pain, especially during second half of night, with improvement on arising
Alternating buttock pain
 Peripheral asymmetric oligoarthritis with predominant large joint involvement – hip, knee,
ankle and shoulder
 Evidence of enthesitis – tenderness and pain at insertion points of muscle, tendon sheaths,
ligaments and fascial planes
 Clinical evidence of dactylitis – this refers to inflammation of the fingers which usually
causes ‘sausage shaped’ digits
 Family history of similar disease/psoriasis or other history suggestive of a cause
 Extra- articular manifestations – uveitis, cardiac involvement
 Good response to NSAIDs
Systemic lupus erythematosus

History

Background

 The patient will usually be a diagnosed patient with SLE, and the focus in the long case will
generally be based on management issues

Presenting complaint

 Make sure you describe the presenting complaint in detail. This will commonly be with a
flare of the disease or with adverse effects of medication

Past history

 Once you have dealt extensively with the presenting complaint, move to the past history
 Focus on the following important aspects
 Discuss the onset of the disease and the basis for the diagnosis – clinical and laboratory
 Ask screening questions to exclude overlap syndromes
 Discuss the initial management and follow up
 Use a timeline to describe the organ complications of the disease

Nervous system complications Episodes of seizures, altered behavior, psychotic


episodes, ask for past history of focal neurological
deficit
Cardiovascular complications Past history of acute coronary events, pericarditis,
valvular lesions
Respiratory Cough and shortness of breath, suggestive of ILD
GI Chronic diarrhea (protein losing enteropathy),
evidence of hepatic involvement (AIH)
Renal Past/current history suggestive of nephrotic/nephritic
syndrome, renal dysfunction
Renal biopsy and immunosuppressive therapy
Hematological Bleeding, past episodes of hemolytic anaemia and
blood transfusions
Malignancy – lymphoma – constitutional symptoms
History of DVT or other episodes of thrombosis
Recurrent pregnancy losses suggestive of APLS

Treatment profiles and drug history

 Discuss the treatment profile of the disease and adverse effects of therapy (see table in the
case of RA)
 Focus on steroid use and related complications
 In addition to this several other immunosuppressive drugs are use d in patients with SLE
 CPP – ask for interruptions in menstrual cycles, sperm or ovum banking, hematological
toxicity, hemorrhagic cystitis
 Azathioprine – liver disease, hematological toxicity
 MMF – hematological toxicity
 HCQ – although not strictly an immunosuppressive drug, this is commonly used in patients
with SLE. Ask the patient on monitoring for retinal toxicity
 Also look for evidence of opportunistic infection

Discuss the functional capacity of the patient

 Use the activities of daily living (ADL) and instrumental activities of daily living (IADL) as a
guide

Past medical and surgical history

Drug history

 This has been described above

Social history

 Describe the personal details of the patient


 Discuss the impact of the disease on the patient on the person and family
 Discuss the support available to the patient
 Describe the household environment with relations to the limitations of activity

Examination

General examination

 Look for the cutaneous manifestations of SLE – these include, malar rash, photosensitivity,
discoid lupus, SCLE, alopecia, oral ulcers (particularly of the hard palate), panniculitis and
vasculitic ulcers
 Look for joint involvement (Jaccoud’s arthopathy)
 Evidence of steroid toxicity – Cushing syndrome
 Lymphadenopathy
 Goitre (autoimmune thyroiditis)
 Edema (lupus nephritis)

Cardiovascular

 Pericardial rubs
 Murmurs (Libmann Sacks endocarditis)

Respiratory

 Pleural effusions
 Evidence of interstitial lung disease

Abdomen

 Organomegaly
Nervous system – for focal neurological signs – hemiparesis, paraparesis
Case summary

A 25-year-old female presents with progressive bilateral ankle swelling, reduced UOP and difficulty in
breathing for 2 weeks duration. She also complains of a low-grade fever for the same duration and
polyarthralgia. Over the last 2 days prior to presentation she has developed oral ulcers and a facial
rash. Examination is significant for a malar rash, bilateral lower limb edema and ulcers in the hard
palate.

How would you establish a diagnosis of SLE?

The classification criteria for SLE was updated in 2012 and may be used to assist the diagnosis. A
diagnosis of SLE is made under the following,

 SLE is likely if the patient satisfies 4 of the clinical and immunologic criteria used in the SLICC
classification criteria, including at least one clinical criterion and one immunologic criterion,
OR
 If the patient has biopsy-proven nephritis compatible with SLE in the presence of ANAs or
anti-dsDNA antibodies
 An ANA titer of > 1:160 is considered significant

What are the major components in the management of SLE?

General management

 A diagnosis of SLE is a major impact on a patient’s life. It is important to counsel the patient
well regarding the diagnosis and implications. Discuss this in detail at the exam
 Discuss pregnancy wishes, family planning and contraception
Pharmacological management

 All patients should be commenced on HCQ unless contraindicated. HCQ has a wide range of
benefits in patients with SLE
 It has been shown to reduce organ damage in SLE and improve survival outcomes. It also
reduces disease activity, has an antithrombotic and lipid lowering effect.

How would you manage lupus nephritis?

 The most important factor in deciding the management of lupus nephritis is the renal biopsy
 Renal biopsy should be should be considered even in asymptomatic patients who have >
0.5g protein excretion per day
 Other general principles on managing glomerular disease should be followed

Renal biopsy findings in lupus nephritis

Management protocols for lupus nephritis

Class Summary of management


Class I No aggressive immunosuppression, general management
Class II Treat as minimal change disease if proteinuria > 3g/24 hours
Class III/IV See below
Class V If persistent proteinuria, treat with steroids and MMF/CP
Class VI No specific therapy, manage as established CKD

Immunosuppressive medication for class III/IV lupus nephritis

 There are two standard protocols for the management of lupus nephritis. Patients with class
III active disease and class IV disease require aggressive immunosuppression. This includes
an induction phase and a subsequent maintenance phase. Induction is usually achieved with
high dose steroids (usually intravenous methylprednisolone and subsequent high dose
steroids (1mg/kg) followed by intravenous cyclophosphamide. Several regimens of CPP are
used.

What are the problems associated with CPP therapy?

 Cyclophosphamide is one of the most potent immunosuppressive medication. Therefore is


has a wide adverse effect profile
 CPP causes major effects on fertility and patients should be counselled regarding this. CPP
can also cause premature ovarian failure. The patient has to be offered options. This
includes sperm/ovum banking, or gonadotropin treatment during CPP therapy, which is
protects the ovaries from premature ovarian failure
 Hemorrhagic cystitis is another major issue with CPP therapy
 Adequate hydration before and after therapy should be practiced. Co-administration of
Mesna is also an option
 Hydration and mesna protocols should be adhered to during administration of CPP

What is the continuing management?

 Start oral prednisiolone at a dose of 1mg/kg and gradually tail off after 3-4 weeks
 Continue 6 cycles of CPP at 2 weekly intervals
 Assess the response of the patient after 3 months. If the patient improves, start
maintenance therapy with MMF and continue for a minimum of 1 year after complete
remission has been achieved

What are the other aspects of management?

 Manage other organ complications – autoimmune hemolytic anaemia, autoimmune


hepatitis
 The management of cerebral lupus may also require high does immunosuppressive therapy
 Consider adverse effects of medication – especially steroids

What are the principles of planned pregnancy in a patient with SLE?

 Pregnancy in SLE should be planned. It requires immense coordination between the


managing medical team, obstetric team and the patient. The general principles are,
 At least 6 months of disease remission
 Consider adjustment of pharmacological therapy – MMF, CYP, CYA, ACEI, warfarin are
contraindicated
 APLS screen if not performed, Anti Ro and Anti La may be arranged. Presence of Anti Ro and
La increase the risk of complete heart block in the neonate
Anaemia

Introduction to the patient

Background – highlight the relevant aspects of the medical background of the patient

The age of the patient is important as it modifies your differential diagnosis

Presenting complaint

The patient will present with shortness of breath and fatigue on exertion with associated lethargy
and dizziness. It is also important to establish whether initial investigations were performed and that
the patient is aware of a low hemoglobin

Description and analysis of the presenting complaint

It is important to give a good chronological description of the presenting complaint and discuss the
symptom profile

Analysis of the presenting complaint

The first step is to decide whether anaemia occurs in isolation or if there is involvement of other cell
lines. Ask for recurrent infections, particularly sino-pulmonary and skin infection, which could
suggest involvement of the white blood cells. Also ask for clinical manifestations of
thrombocytopenia, including petechial rashes, bruising and mucosal bleeding

Proceed with specific questions to establish aetiology

In a patient with pancytopenia,

Pancytopenia

Congenital Age of onset and family history


Myelodysplasia Elderly patient with pancytopenia
Contact/exposure to toxins and chemicals
Lead exposure
Discuss alcohol use
Infection related Ask for past history suggestive of viral hepatitis
Loss of appetite, loss of weight, other manifestations of
tuberculosis
Past history
Neoplastic Associated constitutional symptoms such as low grade
fever, loss of appetite and loss of weight
Immune mediated (SLE related) Ask for symptoms suggestive of autoimmune disease
Low grade fever, inflammatory type joint pain, alopecia
and oral ulcers
Nutritional Dietary history – Vegans (B12 deficiency)
PNH Ask for recent episodes of thrombosis
In the patient with isolated anaemia establish the cause by the following direct questions

Iron deficiency anaemia Ask for episodes of bleeding, malaena, menorrhagia in


females
Ask for upper GI alarm symptoms
Recent onset dysphagia, weight loss, loss of appetite
Obtain a detailed dietary history inquiring specifically into
consumption frequency and amounts of iron rich foods –
meat, fish, milk, eggs and pulses
Highlight the drug history and use of gastric irritant
medication such as NSAIDs and steroids
Highlight the socio-economic profile in the history
Vitamin B12 and folate Ask for associated neurological symptoms including lower
limb weakness, numbness and ataxia
Anaemia of chronic disease Past medical history of chronic disease
Rheumatological disease, renal impairment, chronic heart
failure
Malignancy related Constitutional symptoms
Hemolytic anaemia Associated yellowish discoloration of the eyes with dark
urine (see discussion on Jaundice)
Ask for childhood history of recurrent anaemia requiring
blood transfusion

 After you have done this, establish the patient’s course through healthcare services,
including the investigations and interventions which have been performed
 This may give you a clue to the diagnosis, but do not be biased in your evaluation based on
this
 Evaluate the current functional state of the patient

Past medical history

Dietary history – Obtain a detailed dietary history

Drug history

 Discuss the use of analgesic medication and steroids

Social history

 A detailed social history is paramount


 Discuss the impact of the symptoms on the patient’s activities of daily living and
instrumental activities of daily living
 As mentioned above highlight the patient’s socio-economic living conditions
Examination

General examination

 Pallor
 Icterus
 Look for features suggestive of iron deficiency anaemia – koilonychia, angular stomatitis and
glossitis
 Stigmata of rheumatological disease
 Ankle edema
Cardiovascular system

 Signs of high output failure

Respiratory system

Abdomen

 Look for hepatosplenomegaly

Nervous system

 Look for clinical evidence of subacute combined degeneration of the cord

How would you evaluate anaemia in this patient?

This is a general question in the long case. A prototypic approach to the discussion is presented
below

This is guided by the history and examination. Certain features in the history and examination will
give a clue to the cause of anaemia. Often, the investigations are directed by these findings

 Review the red cell indices and blood picture and classify the anaemia as – microcytic
hypochromic, normocytic normochromic and macrocytic

 The causes of the above morphological forms of anaemia are given in the table below
 Further investigations are based on the morphological type of anaemia

Microcytic Normocytic Macrocytic


MCV < 80 MCV = 80-100 MCV > 100
Iron deficiency anaemia Acute blood loss B12 and folate deficiency
Thalassemia Anaemia of chronic disease Myelodysplasia
Anaemia of chronic disease Bone marrow suppression Hematological malignancies
Less important causes include Hemolytic anaemia Reticulocytosis
Sideroblastic anaemia, lead
poisoning, Cu and Zn
deficiency
Liver disease
Alcohol
Hypothyroidism
Microcytic hypochromic anaemia

The patient should undergo and iron profile. The iron profile interpreted with the blood picture
findings can determine the cause in most cases. Other investigations such as bone marrow biopsy
are required in rare instances. The various types of microcytic anaemias and their diagnostic features
are described in the table below

Diagnosis Serum Iron Ferritin T Sat TIBC Other


Iron deficiency Low Low Low High Blood picture
Microcytic hypochromic cells
Target cells, pencil shaped
cells
Increased RDW
Reduced Retic count
Reduced Marrow Iron
Anaemia of chronic Low High Low Low Increased marrow iron
disease (Low High ESR
transferrin)
Thalassemia trait Normal/High Normal/High Normal/High Normal Blood picture
Many target cells
Normal RDW
High RCC
Thalassemia major High High High Low Blood picture
Many target cells, nucleated
RBC
Normal RDW
High RCC
Sideroblastic anaemia High High High Normal Ring sideroblasts in the
marrow
Lead poisoning (mimics Blood picture
sideroblastic anaemia) Basophilic stippling

Normocytic normochromic anaemia

In patients with normocytic normochromic anaemia a reticulocyte count and retic index is of
significant importance. A high retic count and retic index points towards acute blood loss or
hemolysis. A low retic count/index points towards a marrow production disorder. A bone marrow
aspiration and trephine is required in these patients

Macrocytic anaemia

In patients with macrocytic anaemia a reticulocyte count and index should be performed. A low
count points towards a marrow disorder such as megaloblastic anaemia or myelodysplasia. A high
count is associated with hemolysis

Discussion

Principles of management of anaemia

This question can be asked at any point in your long case discussion. Anaemia is a common
subsidiary problem in the long case
Iron deficiency anaemia The patient should be encouraged to consume a diet rich in
iron containing foods. Locally accessible fish and meat
items can be suggested. Pulses and grains rich in iron
should be recommended. The patient should be advised on
consuming a vitamin C containing source with plant
sources of iron to enhance absorption. Tea contains
phytates that can reduce iron absorption

Iron therapy
Oral
Oral iron is the treatment of choice in the asymptomatic
patient and in most symptomatic patients. Ferrous
sulphate is the most commonly used therapy. Other
preparations such as ferrous fumarate and gluconate have
less adverse effects. Oral iron preparations have adverse
effects such as nausea and vomiting. The Hb increases at an
average of 1g/ per week. Iron should be continued for 3
months after normalizing of Hb to replenish stores. The
target serum ferritin is >50mg/L. A usual course of iron
therapy takes about 4-6 months. Vitamin C may be co-
administered to increase the absorption of oral iron

Parenteral iron
These preparations are indicated in,
Patients who are unable to tolerate oral iron, patients with
malabsorption syndromes, anaemia in pregnancy and
patients with end stage renal disease. It is also useful in
patients who have bowel disease causing reduced
absorption of iron. However these preparations require
strict monitoring during administration as they can cause
anaphylaxis
Commonly used agents are iron dextran and sucrose

Anaemia of chronic disease Treat the underlying disease


See further discussion in the long case of CKD
B12 and folate deficiency Treatment of B12 deficiency – IM hydroxycobalamine (6
doses – 3 -7 days apart). Maintenance therapy can be given
every 3 months

It is important that folate supplementation are given AFTER


vitamin B12 therapy has been commenced. The most
important precursor form of folate is tetrahydrofolate. This
is formed from methyl tetrahydrofolate in the body by the
action of vitamin B12. If B12 is deficient folate is trapped in
the useless form, methyl tetrahydrofolate and cannot be
converted in to the more important precursor forms which
synthesize nucleic acid
Discussion

Case summary

A 23 year old girl presented with progressive shortness of breath, lethargy and tiredness. She had
also noted gradual yellowish discolouration of her eyes and skin. She has not had any similar
episodes before. The patient admits to having regular but heavy menstrual bleeds (lasting for 7 days)
for the last one year. She has had an episode of sore throat, cough and fever 3 weeks ago that
required hospital admission for 4 days. At that time a full blood count was performed and her
haemoglobin was within low normal range. She takes a balanced diet and is not a vegetarian. There
was no family history of haematological disorders, anaemia, still births or consanguineous marriages.
She had been told that her haemoglobin was low on this admission and was being investigated for a
cause.

On examination she was lying comfortably in bed and talked without breathlessness. The
conjunctivae and oral mucosa was pale. A mild jaundice was noted in sclera and skin. There was
palpable cervical, axillary and inguinal lymphadenopathy which was significant. There was no
apparent throat infection. A systolic ejection murmur was noted on precordial examination and the
lungs were clear. There was no hepatosplenomegaly as well.

A case of hemolytic anaemia

Why did you think of a hemolytic anaemia?

 The classical features of a hemolytic anaemia should be present in your history and
examination
 These are, prominent anaemia with yellowish discoloration of the sclerae and
hepatosplenomegaly in the physical examination
 In your long case, it is most likely that these features will occur in the setting of a secondary
disease such as autoimmune disease (SLE), lymphoma. Therefore, keep your eyes open for
any suggestion of these disorders
 Remember in your clinical discussion, that Organomegaly (HSM) is more common in patients
with extravascular hemolysis

How will you investigate this patient?

The initial step in your assessment of the patient is to establish the presence of the hemolytic
anaemia. Discuss this based on the causes described below

Initial evaluation

 Typically cause normocytic normochromic or macrocytic anaemias


 Increased reticulocyte count
 Damaged red blood cells evident on blood picture

Features of red blood cell breakdown

 Elevated serum bilirubin – indirect fraction with increased urobilinogen and without bilirubin
in the urine
 Elevated LDH
 In patients with prominent intravascular hemolysis - decreased serum haptoglobulins,
increased hemosiderin and methalbumin

Features of increased erythropoiesis

 Reticulocytosis
 Bone marrow erythroid hyperplasia and reversal of the myeloid: erythroid ratio

Further investigations should be performed to identify a cause for the hemolytic anaemia

 The Coombs test should be performed to evaluate for the possibility of autoimmune
hemolytic anaemia. The direct Coombs test reveals antibody coated RBC. If this test is
positive, an extended Coombs profile should be performed. If this demonstrates IgG + C3d
specificity it is more likely that the patient has a warm type AIHA
 Cold type AIHA is generally due to IgM antibodies. Further testing of the cold agglutinin
titers can be performed in this case. Discuss that furthermore, cold type AIHA will have only
C3 positivity in the extended Coombs profile with negative IgG and react with RBC when
incubated at low temperatures (< than 30 celsius and optimum at 4 celcius)
 The indirect CT is not generally utilized in the evaluation of an AIHA. This is because that it
examines antibodies in serum
 False positive

A case of hemolytic anaemia in SLE (Warm AIHA)

How would you manage this patient?

 Discuss that you will approach this patient’s management based on several aspects
 Ensure stability of the patient
Consider blood transfusion only if the patient is severely symptomatic. Make sure to
maintain a hemoglobin target of around 8g/dl. Discuss that you will communicate with the
transfusion service and arrange special evaluation for this patient due to the high risk of
transfusion reactions due to RBC antibodies. Rapid transfusion should be avoided, and a
transfusion rate of 1ml/kg/h is reasonable
 The patient should be commenced on folic acid – 5-10mg/d
 Immunosuppression
Steroids are the treatment of choice in these patients
 If a response is still not achieved splenectomy can also be considered. Mention that you will
discuss this option with the patient and the hematologist. If the patient is planned for
splenectomy discuss that you will administer pneumococcal, meningococcal and Hib
vaccines about 2 weeks before the procedure
 Refractory AIHA can also be treated with other immunosuppressive medication such as AZA
 Discuss that you will monitor the adverse effects of therapy (esp. steroids) in subsequent
clinic follow ups

Note – the response of cold AIHA to steroids is very poor. Rituximab has shown some benefit
Approach to a patient with Bruising and bleeding

Case summary

A 26 year old girl presented to ward with gum bleeding. She has had many similar episodes before
over the last two years. Her platelet count has been persistently low over the years even between
episodes. The onset of bleeding was spontaneous and was not precipitated by trauma or fever. There
were no episodes of haematomas forming in muscles or bleeding in to joints. Still, she complained of
a mild occipital headache from yesterday. She is a diagnosed patient with SLE as well. She has been
on several medication including prednisolone and azathioprine for her illness and still has a low
platelet count. The ward doctors have said that her spleen may have to be removed.

On examination there was no visible gum bleeding at the time of interview but multiple purpura of
lower limbs was noted. There was no evidence of large ecchymoses or hemeathrosis. She was not
pale and there was no neck stiffness. The cardiovascular and respiratory systems were clinically
normal and there was no splenomegaly on abdominal examination.

What is your clinical diagnosis?

 When you encounter a patient with a bleeding disorder it is extremely important to localize
the defect based on the pathway of coagulation
 In general disorders of platelet structure and function the patient presents with muco-
cutaneous bleeding manifestations. This includes skin bleeding, oral, nasal mucosal bleeding
and hematuria
 Patients with disorders of the coagulation pathway present with deep bleeding –
hemearthrosis, intramuscular hematomas and life threatening organ bleeding
 The long case can feature a patient with autoimmune disease with thrombocytopenia,
suggesting a diagnosis of ITP

How would you confirm the diagnosis of ITP?

 Discuss that the diagnosis of ITP is one of exclusion


 The FBC will demonstrate isolated thrombocytopenia with a blood picture demonstrating
large platelets
 A bone marrow examination will demonstrate increased megakaryocytes
 Anti-platelet antibody assays are not important in the diagnosis as they do not have
adequate sensitivity and specificity
 It is important to plan out investigations to look for underlying secondary causes of ITP.
Discuss that you would request an ANA, retroviral screening test and look actively for the
possibility of a lymphoma. Presence of a splenomegaly on examination may suggest a
secondary cause such as a lymphoma or SLE

What are the principles of management in this patient?

 Patient education is an important aspect of the management


 Pharmacological therapy
 The decision to commence pharmacological management is based primarily on the
symptoms of the patient and the platelet count
 If the patient is asymptomatic with a platelet count of more than 30,000 no f urther therapy
is required and the patient can be observed
 Steroids are the mainstay of management. Oral prednisolone 1mg/kg is the most commonly
used therapy

How will you manage a patient who presents with massive intra-abdominal bleeding?

 However if the patient has severe thrombocytopenia (platelet count of < 5000) or life
threatening bleeding manifestations urgent steroid pulses and IVIG should be commenced.
Discuss that you may consider platelet transfusions in dire need. Anti D immunoglobulin is
an alternative to IVIG in patients who are rhesus positive

Continuing management

 If the patient responds to therapy start to gradually tail off after about 4 weeks
 AZA is commonly used as steroid sparing therapy
 Treatment options for refractory ITP include splenectomy, rituximab therapy and TPO
receptor agonists. All options are available in Sri Lanka
 Specialized management is required in females of reproductive age, as antiplatelet
antibodies can cross the placenta, they can cause fetal thrombocytopenia
Lower limb weakness

The clinical approach to analysis of lower limb weakness

The clinical approach to lower limb weakness requires information from both the history and the
examination

Presenting complaint

The patient presents with lower limb weakness

Description of the presenting complaint

 Make sure that you do an extensive description of the presenting complaint in chronological
sequence
 Describe the onset, evolution and progression of the symptom
 Ask about the associated symptoms

Analysis of the presenting complaint

 The first step is to describe the pattern of weakness – hemparesis, paraparesis,


quadriparesis and localized weakness
 The next step is to decide whether the weakness is upper motor neuron type or lower motor
neuron type
 Perform the neurological examination at this point
 The following table gives the cardinal features of differentiation

Upper motor neuron Lower motor neuron


Wasting Minimal wasting in Wasting in proportion with
consideration with the degree of the degree of disability and
weakness and disability weakness
Fasciculation None Noted
Tone Elevated and is of clasp knife Reduced (flaccid)
quality
Pattern of weakness Flexors are generally weaker Variable pattern of weakness
than extensors
Reflexes Brisk reflexes Diminished or absent reflexes
Clonus
Extensor plantar response
Loss of superficial abdominal
reflexes

Further analysis

1. Lower motor neuron lesion – Paraparesis and quadriparesis and localized weakness

A lower motor neuron lesion could mean a lesion at any point along the lower motor neuron
pathway. The following table describes cardinal features at each site which can be used for
differentiation
History Examination Further questions for
aetiology
Muscle - myopathy Can be acute or Bilateral, proximal > distal Ask for family history
subacute in nature weakness Features of connective
The patient will have Minimal/no wasting tissue disorders, skin rashes
prominent difficulty in No fasiculations Drugs (statins)
standing up from the Preserved deep tendon Features of hypothyroidism
seated position, reflexes
climbing stairs and No sensory loss
combing hair or Muscle tenderness in the case
brushing teeth of myositis
Muscle pain may also
be a feature
NMJ Is usually subacute in No wasting/fasiculations Ask for other features of
onset and progression Preserved deep tendon autoimmune disease
The weakness reflexes
fluctuates throughout No sensory loss
the day and is altered Fatigability (MG)
by activity
Cranial muscle
involvement is usually a
dominant feature
Neuropathy
Mononeuropathy Acute or subacute Motor and sensory signs are Trauma, compression
presentation localized to a single nerve
Neuropathy
Polyneuropathy Acute or subacute Distal > proximal weakness Ask for preceding diarrhea/
presentation noted in most (axonal chest infection (GBS)
Commonly has a length neuropathy) Nutritional history
dependent progressive Proximal = Distal or Proximal > History of autoimmune
distribution Distal noted in demyelinating disease (SLE)
Sensory symptoms neuropathy Constitutional symptoms,
Glove and stocking sensory fever, non length dependent
loss neuropathy (paraneoplastic,
HIV)
Backache (Myeloma,
POEMS)
Neuropathy
Mononeuritis multiplex Patchy neural Patchy involvement Ask for features suggestive
involvement of systemic vasculitis
Root Acute or subacute Myotomal and dermatomal
presentation distribution of neurological
Associated radicular features
pain Involvement of reflexes is
based on myotomal supply
Plexus Multiple mytomal and
dermatomal involvement in a
contiguous manner
AHC Chronic, slowly Asymmetrical involvement,
progressing symptoms commencing from proximal
muscles
Significant wasting and
fasciculations
Mixed upper motor and lower
motor signs
No bladder or bowel
involvement
No sensory loss
Spinal cord (spinal shock) Acute or subacute Symmetrical or asymmetrical
progressing symptoms involvement
Usually presents with
paraparesis or quadriparesis
Early bladder and bowel
involvement
Sensory level

2. Upper motor neuron lesion – paraparesis and quadriparesis


This usually indicates a lesion in the spinal cord, or rarely the brainstem. A parasaggital
meningioma can rarely cause paraparesis

History Examination Specific questions for


aetiology
Brainstem lesion Acute or subacute Cranial nerve palsies
progression With quadriparesis
Altered consciousness in
most cases
Cranial nerve
dysfunction
Cervical cord lesion Early bladder and bowel Quadriparesis Cord lesions
dysfunction Cord syndromes (anterior, Ask for history of trauma
Sensory level Central) Ask for significant bone
pain, sinister backache,
constitutional symptoms
(neoplastic, infective)
Ask for preceding infection
(TM), autoimmune disease
Ask for associated visual loss
(NMO/MS)
Sudden onset (with anterior
cord syndrome)
Thoracic cord lesion Paraparesis As above
3. Upper motor neuron lesion – hemiparesis (see separate discussion on stroke)
Hemiparesis usually indicates a lesion in the cortex or the internal capsule

Dominant hemispheric Sudden onset indicates a Hemiparesis


lesion vascular event Associated hemisensory loss
Progressive/subacute can Homonymous hemianopia
indicate demyelination or Dysphasia
mass lesion Dyscalculia and agraphia
Finger agnosia and left/right disorientation
Non dominant As above Hemiparesis
hemispheric lesion Associated hemisensory loss
Homonymous hemianopia
Visuo-spatial disorientation
Hemispatial neglect
Constructional and dressing apraxia
Brain stem lesion As above Cranial nerve palsies
Vertigo
Diplopia
Dysphagia with nasal
regurgitation
Internal capsule Pure motor
Mild dysarthria

Social history

 A detailed social history in a patient with neurological symptoms is paramount


 Discuss the impact of the symptoms on the patient’s activities of daily living and
instrumental activities of daily living
 Discuss issues of accessibility
 Explore the psychosocial issues of the patient in detail

Discussion on peripheral neuropathy

How would you approach a patient with peripheral neuropathy?

 This is based on the clinical pattern of the neuropathy


 The clinical pattern is described as,
Mononeuropathy, Polyneuropathy or MNM
Length dependent or length independent
Sensory, motor or mixed
Autonomic involvement
 Once the clinical pattern of the neuropathy is established in the clinical evaluation a nerve
conduction study should be requested
 Further investigations should be performed based on the findings of the nerve conduction
study
Guillain- Barre syndrome

A 25 year old previously healthy lady presents with progressively ascending weakness and difficulty in
breathing. She admits to have had symptoms of gastroenteritis 3 weeks back. Her neurological
examination is significant for B/L lower limb and upper limb weakness and areflexia. Her sensory
system is intact.

What is the most likely diagnosis?

Guillain- Barre syndrome

How are you going to manage this patient?

The most important aspect of the management is the monitoring and regular assessment of the
respiratory capacity of the patient. This can be done by single breath counting test or more
objective assessment by FVC.

 Cardiac monitoring is also required as the patient can have autonomic instability which
manifests as fluctuating BP, bradycardia and arrhythmias.
 ICU care is preferred
 Admission to the ICU should be considered for all patients with labile dysautonomia, an FVC
of less than 20 mL/kg or severe bulbar palsy
 Definitive care is provided by plasma exchange or IV immunoglobulin. Both these are proven
to have same efficacy.
 Prevent complications due to prolonged immobilization
 Rehabilitation

What is the CSF abnormality seen in patients with GBS?


CSF analysis is not conclusive until 10 days. The typical pattern is increased protein level with no
increase in the cell count. (Albuminocytologic disassociation)

Pathophysiology and various subtypes of GBS.

 GBS includes at least 4 subtypes of acute peripheral neuropathy


 AIDP
Mediated by T cells against peptides from myelin proteins
 AMAN
 AMSAN
Antibodies to gangliosides on the axonal membranes
 Miller- Fischer syndrome
A young patient with stroke

Background and introduction

Presentation and description of the presenting complaint

 Discuss the presenting complaint in a chronological sequence. This is especially important in


neurology. Describe the onset and progression of the neurological manifestations in detail

Analysis of the presentation

 Use the clinical presentation to localize the lesion (see above)


 Exclude the possibility of other stroke mimics
CNS infection with focal neurological manifestations – ask for fever with signs of meningism
and altered behavior
Bleeding into a tumor – Recent history of headache with features of increased intracranial
pressure
Demyelinating disorders – Past history of similar focal neurological symptoms with
localization to other parts of the nervous system including the optic nerves and spinal cord
Hemiplegic migraine – typical headache suggestive of migraine with transient weakness
Post ictal Todd’s paresis – History of seizures
Hypoglycemia presenting with focal neurological deficit – typical symptoms of
hypoglycaemia
 Discuss the aetiology of the lesion

Atherothromboembolism Ask for risk factors for atherosclerosis


Cardioembolic stroke Ask for past history of cardiac disease, cardiac failure,
valvular and congenital heart disease
Past history of rhythm disorders, especially atrial
fibrillation
History suggestive of infective endocarditis
Vasculitis Ask for history of large vessel vascular disorders
TA – arm claudication
History of vasculitis, joint pain with inflammatory nature,
skin rash, alopecia
Multisystem involvement – involvement of other vascular
territories
Past history of autoimmune disease - SLE
PANCNS
Reversible cerebral vasoconstriction – thunderclap
headache preceding neurological manifestations
Infection related vasculitis
Use of illicit drugs, risk factors for retroviral illness
Thrombotic disorders Past history of arterial/venous thrombosis, stroke in the
setting of autoimmune disease
Family history of thrombotic disorders
Medication history – use of OCP
Hematological disorders Family history of hemoglobinopathy
Possible myeloproliferative disease
Rare genetic disorders CADASIL and MELAS – family history
Vascular dissection Recent history of trauma, other evidence of collagen
disease
Intracranial hemorrhage AVM, tumors
Use of anticoagulation disorders

Complete the other components of the history

Social history

 A detailed social history in a patient with neurological symptoms is paramount


 Discuss the impact of the symptoms on the patient’s activities of daily living and
instrumental activities of daily living
 Discuss issues of accessibility
 Explore the psychosocial issues of the patient in detail

Examination

General examination

 Pallor or plethora
 Icterus
 Cutaneous features of autoimmune disease
 Evidence of collagen disorders
 Clubbing
 Cutaneous evidence of hyperlipidaemia
 Ankle edema
Cardiovascular disease

 Assess the cardiac rhythm


 Evaluate for murmurs and cardiac structural abnormalities
 Pulmonary hypertension
Respiratory

Abdomen

 Organomegaly may indicate hematological disorders – PRV/ET with thrombotic


manifestations

Nervous system

 Perform a detailed evaluation of the nervous system


 Fundus
 Clinical localization of the lesion
Case summary

A 59-year-old male presents with sudden onset R/S face, arm and leg weakness with d ifficulty in
speaking for 2 hours duration. He has a background of hypertension and dyslipidaemia. Examination
is significant for a blood pressure of 190/110 and bilateral carotid bruits. Neurological examination
reveals expressive dysphasia, R/S upper motor neuron type facial nerve palsy, with upper motor
neuron type weakness in the R/S upper and lower limbs

How would you manage this patient in the acute stage?

 Stroke is a medical emergency. Following the clinical diagnosis of stroke prompt attention
should be focused on airway stabilization and optimization of the circulation. The latter is
more important in patients presenting with posterior circulation syndromes.
 Oxygen should be administered if the saturation of admission is less than 94%
 It is important to remember that the management of stroke is an extremely time dependent
process and an extra effort should be taken to expedite the following investigations. CT
brain may not be available in all hospitals in Sri Lanka and early transfer is required

Imaging - Non contrast CT scan of the brain

 An urgent non contrast CT of the brain should


be requested for exclusion of a hemorrhagic
lesion. The CT May be normal in ischaemic
stroke especially up to 24 hours since onset of
symptoms. Initial changes may show up
between 3 – 24h and within 3h in a large
infarct
Changes include hypodensity and loss of the
grey-white demarcation. An early sign of MCA
infarction is known as the hyperdense MCA
sign

Is MRI superior for the diagnosis of stroke?

 MRI is superior for the diagnosis of lacunar


and posterior circulation infarcts but a standard MRI can miss cerebral hemorrhage in the
hyperacute stage. Discuss that in addition, other issues such as accessibility to MRI and time
taken for image acquisition also make CT a better option for the diagnosis

Discuss the role of reperfusion therapy in this patient

 The patients should be assessed for reperfusion. Early reperfusion improves patient
outcomes. Reperfusion therapy includes therapies to re-establish the cerebral blood flow.
The current options available for reperfusion include, intravenous rtPA and endovascular
reperfusion. Both options are available in Sri Lanka, but endovascular reperfusion is only
available in the private sector.
Intravenous rtPA

 Intravenous rtPA is available at the NHSL Colombo for stroke management. The agent used
is alteplase. This drug activates tissue plasminogen, which converts plasminogen to plasmin,
which subsequently causes clot lysis
 A formal assessment of the inclusion and exclusion criteria should be performed
 The current inclusion window is within 4.5 hours since symptom onset. However the best
time window for rtPA is probably 3 hours from symptom onset

Inclusion and exclusion criteria for rtPA

 The patient should have a clearly defined symptom onset and a measurable neurological
deficit at the time of assessment. Formal
evaluation may be performed using the
NIHSS score
 If the patient presents between 3h-4.5h
following stroke the additional inclusion
criteria should be met.
 Age less than 80 years
Not on anticoagulant therapy
No past history of stroke together with
DM
NIHSS score less than 25
Infarcted area less than 1/3 of the MCA
territory
 If the patient is suitable for rtPA, all
exclusion criteria should be examined
before administering the drug.

Administration of rtPA

 The target time for administration of rtPA to suitable patients should be within 60 minutes
of presentation to hospital
 If rtPA is administered perform frequent blood pressure monitoring, keep the patient off all
other antithrombotic therapy for 24 hours
 Monitor the patient neurologically for decline in consciousness and new onset neurology.
This could be suggestive of an intracranial hemorrhage

Antithrombotic therapy

 Aspirin is the only antiplatelet agent proven to be effective in the management of patients
with acute ischaemic stroke and should be commenced within 48 hours of stroke onset. The
recommended dose is 300mg. This should be continued up to a maximum of 2 weeks. If the
patient has been treated with rtPA the initial aspirin dose should be delayed by 24 hours
 Oral/IV anticoagulants do not have a role in the acute management of stroke and should not
be commenced
 A combination of aspirin and clopidogrel has been found to be useful in minor strokes
Supportive medical management

 Nurse on a side to prevent aspiration


 NG feeds if swallowing is impaired or patient is drowsy
 If the patient is unconsciouses or drowsy, manage appropriately with care to prevent
pressure sores
 Catheterize only if incontinent. Unnecessary catheterization can result in urinary tract
infections
 BP management in acute stroke is controversial. Hypertension should be managed in
patients with BP = 220/ and or 120 not undergoing thrombolysis and 185/ and or 110 in
patients undergoing thrombolysis
The blood pressure should be closely monitored in all patients with stroke. Uncontrolled
blood pressure can cause hemorrhagic transformation. Rapid decline in blood pressure can
increase the infarct size
It is important to manage elevated blood pressure with intravenous antihypertensives. This
is because these drugs have a very short half – life and can be titrated very well. Of the drugs
available in Sri Lanka, labetalol is probably the best to use. IV GTN can cause cerebral
vasodilation and more intracerebral bleeding complications
 Manage fever and treat all infections promptly as hyperthermia can increase cerebral
damage
 Manage hyper and hypoglycaemia
It is reasonable to maintain the blood glucose from 140-180 mg/dl
 Special attention should be given to patients with worsening cerebral edema following
stroke. Mannitol should be administered in these patients and there is a role for
decompressive craniotomy, if available. Steroids should not be used for management of
increased ICP in patients following stroke

Stroke unit

 Patients with stroke should be managed in a specialized stroke unit with availability of a
multidisciplinary team including physiotherapists, occupational therapists and social care
workers
 Even if a stroke unit is not available in the hospital all patients with stroke should have a
swallowing assessment performed as soon as possible and the route of feeding decided
 The patient should be targeted for early mobilization
 DVT prevention should also be looked into in patients who are immobile
 Every effort should be made to make the patient function independently and to reintegrate
him/her into society

How would you evaluate the patient after the initial stage?

 The primary objective of this evaluation is to identify the aetiology for the stroke
 Basic investigations for evaluation of atherosclerotic risk factors should be performed – Lipid
profile, fasting blood glucose
 Evaluate the patient for atrial fibrillation
 Request for an echocardiogram – to look for structural cardiac disease including valvular,
congenital heart disease and PFO. The echo will also reveal evidence of endocarditis
 Discuss that you will evaluate the patient for the rare causes of stroke – autoimmune profile,
coagulation profile thrombophilic screen – APLS, protein C and S, Factor V Leiden mutation,
AT111 mutation, homocysteine levels

Discuss the continuing management of this patient

 The continuing management of this patient will be multifaceted


 Rehabilitation
 Secondary prevention – Blood pressure management, management of hyperlipidaemia,
antiplatelet agents
 Anticoagulation in patients with AF and stroke – this is generally commenced 2 weeks after
the stroke (especially if it involves a large area) due to the risk of hemorrhagic
transformation. Anticoagulation can be commenced earlier in patients with minor stroke

What antiplatelet agents will you use in this patient?

 Based on current evidence


 Clopidogrel monotherapy has a marginal increase in benefit when compared to aspirin. It is
also non-inferior to combination therapy with aspirin and ER dipyridamole
 Aspirin and clopidogrel combination therapy has been shown to be beneficial in patients
with minor stroke when continued up to a duration of 3 months
 Combination therapy with aspirin and extended release dipyridamole. However this
combination is not available in Sri Lanka and headache is a common complication of
dipyridamole therapy

You might also like